SlideShare una empresa de Scribd logo
1 de 27
Pediatric Dermatology
1) A full term newborn develops erythematous, indurated plaques on the upper back. Which of the
following tests should be performed?


A. Thyroid function test

B. CalciumCorrect Choice

C. Blood culture

D. Complete blood count

E. Alkaline phosphatase
The most likely diagnosis is subcutaneous fat necrosis of the newborn. Hypercalcemia may result
and serum calcium levels should be monitored up to 4 weeks after resolution of the skin


2) What is the most common tumor associated with this condition?

A. Trichoadenoma

B. Trichoblastoma Correct Choice

C. Syringocystadenoma papilliferum

D. Basal cell carcinoma

E. Sebaceous carcinoma
Recent reviews have identified trichoblastoma as the most common tumor arising within nevus
sebaceous


3) Which finding is associated with Pachydermoperiostosis:

A. Mucosal keratoses

B. Cutis marmorata

C. Cutis verticis gyrataCorrect Choice

D. Palmoplantar hyperkeratosis

E. Osteopathia striata
The scalp change seen in pachydermoperiostosis is cutis verticis gyrata


4) An infant presents with the lesion depicted in the photo. What information do you provide the
parents regarding this condition?


A. The lesion will persist and may grow further Correct Choice

B. The lesion is associated with a XO karyotype

C. The lesion will respond to laser therapy

D. The lesion will go through rapid growth followed by stabilization and regression

E. The lesion will resolve with antifungal therapy
The lesion represented in the photo is an arteriovenous vascular malformation. These lesions do not
typically regress


5) Which of the following is most likely associated?


                                                     1
A. No additional abnormalityCorrect Choice

B. Deafness

C. Bifid rib

D. Alopecia

E. Coarctation of the aorta
The majority of patients with accessory tragii do not have any associated abnormalities. Multiple
anomalies, including deafness and midline defects, have been reported but are rare
Trichothiodystrophy classically includes which of the following findings:


A. Trichorrhexis nodosa

B. Pili torti

C. Trichostasis

D. TrichoschisisCorrect Choice

E. Trichorrhexis invaginata
Trichothiodystrophy or PIBIDS is associated with the “tiger-tail” abnormality of trichoschisis


7) A 3 month-old girl with multiple hemangiomas along her right jaw is at increased risk for:

A. Subglottic hemangiomaCorrect Choice

B. Hearing defecits

C. Underlying bone abnormalities

D. Bleeding complications

E. Oral obstruction
Hemangiomas are benign vascular tumors which have proliferating phase and then a spontaneous
involution phase. Depending upon the location, some hemangiomas may have more long term
sequelae such as scarring or structural malformation, and rarely, consumptive coagulopathy.
Infants with hemangiomas of the “beard” distribution should be evaluated for subglottic
hemangiomas which may lead to airway obstruction
A 3 month-old girl with multiple hemangiomas along her right jaw is at increased risk for:


A. Subglottic hemangiomaCorrect Choice

B. Hearing defecits

C. Underlying bone abnormalities

D. Bleeding complications

E. Oral obstruction
Hemangiomas are benign vascular tumors which have proliferating phase and then a spontaneous
involution phase. Depending upon the location, some hemangiomas may have more long term
sequelae such as scarring or structural malformation, and rarely, consumptive coagulopathy.
Infants with hemangiomas of the “beard” distribution should be evaluated for subglottic
hemangiomas which may lead to airway obstruction


8) In a child with zinc deficiency, yet normal or near normal zinc levels, which test could be a
valuable adjunctive test?


A. Niacin




                                                  2
B. Iron

C. Alkaline phosphataseCorrect Choice

D. Magnesium

E. Manganese
The diagnosis of zinc deficiency should be consideredi in at-risk individuals with acral or periorificial
dermatitis. Chronic diaper dermatitis in an infant should lead to the evaluation for zinc deficiency. A
low serum zinc level can usually confirm the diagnosis. If normal or near normal a low serum
alkaline phosphatase, a zinc-dependent enzyme, may be a valuable adjunctive test


9) Posterior auricular adenopathy is a common feature of which exanthem:

A. Rubella Correct Choice

B. Scarlet fever

C. Measles

D. Erythema infectiosum

E. Mumps
Lymphadenopathy with 1-7 days of malaise is commonly described with rubella infection. The rose-
pink macular rash follows the prodrome


10) What is best the test to confirm a diagnosis?

A. Anti Ro antibody

B. Biopsy

C. Gram stain

D. KOHCorrect Choice

E. NA antibody
The picture depicts tinea faceii. A KOH exam should be performed to look for hyphae


11) What is the diagnosis?

A. Neonatal cphalic pustulosisCorrect Choice

B. Transient neonatal pustular melanosis

C. Erythema toxicum neonatorum

D. Acropustulosis of infancy

E. Miliaria
In neonatal cephalic pustulosis (neonatal acne), papules and pustules are present but comedones
are absent. Spontaneous remission occurs


12) Which of the following is a potential complication of subcutaneous fat necrosis of the newborn?

A. Cholestrol clefts in fat cells

B. Acute renal failure

C. Hepatitis




                                                    3
D. HypercalcemiaCorrect Choice

E. Elevated uric acid levels
Subcutaneous fat necrosis of the newborn is a rare condition characterized by firm, reddish or
purple nodules which appear on the arms, thighs, buttocks, back and cheeks. This condition is
believed to result from cold injury. Crystallization occurs within the lipocytes, and this can be seen
on histopathology with an associated granulomatous reaction. This is thought to occur due to the
higher melting point of neonatal fat, which contains more saturated fatty acids. Onset of this
condition occurs within the first two weeks of life and usually resolved over a period of weeks to
months. Occasionally, lesions heal with atrophy. Hypercalcemia is an infrequent complication of
subcutaneous fat necrosis of the newborn. Associated symptoms can include irritability, weight loss,
vomiting and failure to thrive. Repeated serum calcium tests are advised until one month after all
the cutaneous lesions have resolved


13) What is the most appropriate management of this condition?

A. Observation Correct Choice

B. Pulsed dye laser

C. Embolization

D. Oral corticosteroids

E. MRI
This V-shaped vascular lesion is nevus flammeus neonatorum. Most lesions will fade over time
without intervention


14) An infant with failure to thrive has multiple xanthomas on skin exam and foamy histiocytes on
bone marrow biopsy. Your diagnosis is:


A. Niemann-Pick diseaseCorrect Choice

B. Fabry’s disease

C. Tay-Sach’s disease

D. Hunter’s syndrome

E. Gaucher’s disease
Niemann-Pick disease is an autosomal recessive disease caused by mutations in sphingomyelin
phosphodiesterase-1. Patients with Type A Niemann-Pick disease may have xanthomas, progressive
psychomotor deterioration, hepatosplenomegaly, blindness, cherry red spots, and deafness


15) An infant presents with yellowish-brown, crusted papules with petechiae in a seborrheic
distribution. Which of the following statements about this entity is likely to be true?


A. Maternal-fetal transmission most likely occured in the peripartum period

B. This presentation occurs when the infant is weaned off of breast milk

C. CD1+, S100+ cells with comma-shaped nuclei should be seen on biopsyCorrect Choice

D. An autosomal recessive defect in holocarboxylase synthetase is the cause

E. This presentation is consistent with Jacquet's dermatitis
This infant has Langerhans Cell Histiocytosis (Letterer-Siwe disease). Multisystem involvement may
be present. Jacquet's Erosive Dermatitis presents with severe erosive papules in the diaper region,
and is multifactorial in etiology (yeast, irritant dermatitis, moisture). Acrodermatitis enteropathica
(zinc deficiency) presents with brown, orange crusted plaques with vesicles and bullae, especially in




                                                   4
perineal and perioral areas and distal extremities. The inherited form occurs when the infant is
weaned off of breast milk. Biotin deficiency presents similarly; the neonatal form is attributed to an
AR defect in holocarboxylase synthetase


16) What is the best therapeutic option?

A. Topical mupirocin

B. Oral prednisone

C. Topical immune modulator

D. Oral cephalexin

E. Oral acyclovirCorrect Choice
The condition shown in an example of eczema herpeticum, also called Kaposi varicelliform eruption.
It occurs when pre-existing dermatitis becomes superinfected with a viral infection, most commonly
atopic herpes infection of atopic dermatitis. The initial treatment of choice is antiviral medications


17) Multiple cylindromas are associated with:

A. Trichoepitheliomas Correct Choice

B. Cowden syndrome

C. Pilomatrichomas

D. Carney complex

E. Myotonic dystrophy
The Brooke-Spigler syndrome is defined by the presence of multiple trichoepitheliomas and
cylindromas


18) This condition may be associated with which of the following:

A. Perinatal trauma

B. Hyperlipidemia

C. Hypercalcemia

D. Epidermolysis bullosa dystrophica Correct Choice

E. Vitiligo
Aplasia cutis congenita may be associated with dystrophic epidermolysis bullosa (Bart syndrome


19) Neutrophils are characteristically seen on smears from pustules of which of the following
transient benign neonatal rashes?


A. Acne neonatorum

B. Erythema toxicum neonatorum

C. Acne neonatorum and transient neonatal pustular melanosisCorrect Choice

D. Infantile acne

E. Transient neonatal pustular melanosis
Neutrophils are seen on smears from pustules of acne neonatorum and transient neonatal pustular
melanosis. Eosinophils are characteristically seen on smears from erythema toxicum neonatorum




                                                  5
20) Which of the following is characteristic of Wooly Hair Nevus:

A. Ocular abnormalities may be associated

B. Sponteneous improvement never occurs

C. Typically occurs in the 5th decade

D. There are no nevi associated with this hair abnormality

E. It is hereditaryCorrect Choice
Wooly Hair Nevus is the presence of Negroid hair on the scalp of persons of non-Negroid
inheritance. The unruly hair presents at birth or in infancy, usually as a solitary problem inherited in
an autosomal dominant fashion


21) What is the best treatment option?

A. Topical tretinoinCorrect Choice

B. Oral acyclovir

C. Topical ketoconazole

D. Observation

E. Oral cephalexin
The lesions shown in the photo are comedones of infantile acne. Topical tretinoin would be the best
treatment option


22) Which of the following can present as collodion baby?

A. X-linked ichthyosis

B. Bullous congenital ichthyosiform erythroderma

C. Sjogren-Larsson syndrome

D. Lamellar ichthyosisCorrect Choice

E. Ichthyosis vulgaris
The most common presentation of collodion baby is lamellar ichthyosis, followed by congenital
ichthyosiform erythroderma. Patients with ichthyosis vulgaris and x-linked ichthyosis are normal at
birth. Bullous congenital ichthyosiform erythroderma or epidermolytic hyperkeratosis presents with
widespread bullae, erthroderma, and denuded skin. Sjogren-Larsson presents with generalized
ichthyosis and erythroderma in infancy. It is important to know the at-birth presentations of all the
disorders of cornification


23) A 2 day-old full term neonate develops blotchy erythematous macules with small central
pustules over the upper trunk and extremities. A gram stain reveals predominantly eosinophils.
What is the most likely diagnosis?


A. Incontinentia pigmenti

B. Erythema toxicum neonatorum Correct Choice

C. Urticaria pigmentosa

D. Transient neonatal pustular melanosis




                                                   6
E. Miliaria
Erythema toxicum neonatorum is a very common eruption in healthy newborns. A gram stain
reveals sterile pustules containing eosinophils


24) An eight year-old boy presents with pink, flushed cheeks and a low-grade fever. The following
eruption then appeared. What is the most likely etiology?


A. Paramyxovirus

B. Togavirus

C. Parvovirus Correct Choice

D. HHV6

E. Group A streptococcus
This condition, also called Fifths Disease or erythema infectiosum, is caused by parvovirus B19, a
single-stranded DNA virus. There are a few clinical presentations including, diffuse lacy rash on the
trunk that spreads gradually toward the distal extremities, papular-pruritic "gloves-and-socks"
syndrome and aplastic crisis which does not have concomitant rash


25) What is the best test to confirm a diagnosis?

A. Gram stain

B. KOH Correct Choice

C. Biopsy

D. Anti Ro antibody

E. DsDNA antibody
The picture depicts tinea faceii. A KOH exam should be performed to look for hyphae


26) Which presentation of psoriasis is more common in children:

A. Acrodermatitis continua of Hallopeau

B. Guttate psoriasisCorrect Choice

C. Keratoderma blennorragica

D. Pustular psoriasis

E. Erythrodermic psoriasis
The majority of cases of guttate psoriasis occur in persons under the age of 30


27) This patient had significantly elevated serum CPK. The likely diagnosis is:

A. Dermatomyositis Correct Choice

B. Lupus erythematosus

C. Atopic dermatitis

D. Psoriasis

E. Lichen planus




                                                    7
The answer is dermatomyositis with the heliotrope color and distribution of erythema or violaceous
color. The skin over the metacarpal and proximal interphalangeal joints can become inflamed and
erythematous forming Gottron’s papules


٢٨) Menkes kinky hair syndrome is associated most commonly with:

A. Trichorrhexsis invaginata

B. Pili torti Correct Choice

C. Pili multigemini

D. Plica neuropathica

E. Trichostasis spinulosa
The X-linked recessive Menkes kinky hair syndrome is associated with multiple hair shaft
abnormalities, most characteristically, pili torti


29) What disorder is associated with a defect in LMX1B?

A. Lhermitte-Duclos syndrome

B. Chediak-Higashi syndrome

C. Steatocystoma

D. Nail patella syndrome Correct Choice

E. Monilithrix
A LMX1G gene defect is associated with nail patella syndrome


30) Schimmelpenning-Feuerstein-Mims syndrome may be associated with which of the following:

A. Hypophosphatemic ricketsCorrect Choice

B. Polyostotic fibrous dysplasia

C. Osteopathia striata

D. Chondrodysplasia punctata

E. Osteopokilosis
Epidermal nevus syndrome, also known as Schimmelpenning-Feuerstein-Mims syndrome, may be
associated with hypophospatemic, vitamin D resistant rickets


31) Variants of xeroderma pigmentosum are due to all of the following defects except:

A. Endonuclease

B. Helicase

C. Thymidine kinase Correct Choice

D. Postrepliction repair

E. Nucleotide excision repair
The photosensitivity of xeroderma pigmentosum is caused by defect in DNA repair mechanisms.
Thymidine kinase is not affected


32) “Slapped cheeks” followed by a lacy eruption on extremities:


                                                 8
A. ssRNA virus

B. dsDNA virus

C. ssDNA virus Correct Choice

D. Streptococcus

E. dsRNA virus
The erythematous eruption on the cheeks precedes the generalized lacy rash of Fifth disease. The
causative agent is parvovirus B19, a single stranded DNA virus


33) Ankyloblepharon filiforme adnatum is seen with a defect in:

A. p63 Correct Choice

B. Plakophilin

C. Desmoglein

D. C-kit

E. Plakoglobin
The ectodermal-clefting syndromes are caused by a defect in the p63 gene. Specifically, AEC or
Hay-Wells syndrome is comprised of ankyloblepharon filiforme adnatum, ectodermal dysplasia and
clefting


34) Pachyonychia congenita type 2 is most commonly associated with which of the following:

A. Deafness

B. Aplastic nails

C. Increased risk of malignancy

D. Natal teethCorrect Choice

E. Poikiloderma
Pachyonychia congenita type 2 may be associated with natal teeth and steatocystoma.
Pachyonychia congenita type 1 is associated with benign leukoplakia


35) The risk of fetal death with intrauterine parvovirus infection may occur with infection in which
trimester:


A. First

B. Third

C. Second

D. First, Second and Third Correct Choice

E. None of these answers are correct
Fetal hydrops may occur with parvovirus infection during all three trimesters although the greatest
risk is during the second trimester. Congenital anomalies are not a feature


36) Individuals with which of the following syndromes characteristically present with
photosensitivity, mental retardation, a "wizened" appearance, "bird-headed" facies, and "Mickey
Mouse" ears?




                                                  9
A. Rothmund-Thomson Syndrome

B. Werner Syndrome

C. Tay Syndrome

D. Cockayne SyndromeCorrect Choice

E. Hutchinson-Gilford Progeria Syndrome
Cockayne Syndrome is AR, caused by defective excision repair, cross-complementing group 8 gene
(ERCC8). It presents with photosensitivity, mental retardation, and cachectic dwarfism. Patients
have a characteristic "wizened" appearance, "bird-headed" facies, and "Mickey Mouse" ears.
Cataracts, deafness, pigmentary retinopathy, dental caries, and skeletal, GU, and endocrine
abnormalities may be seen


37) What is the most common tumor associated with this condition?

A. Sebaceous carcinoma

B. Trichoadenoma

C. Trichoblastoma Correct Choice

D. Basal cell carcinoma

E. Syringocystadenoma papilliferum
Recent reviews have identified trichoblastoma as the most common tumor arising within nevus
sebaceous


38) What the most likely diagnosis?

A. Wiskott-Aldrich syndrome

B. Atopic dermatitis Correct Choice

C. Rud syndrome

D. Lamellar ichthyosis

E. Keratosis pilaris
Ichythosis vulgaris and hyperlinear palms are both independent minor criteria for the diagnosis of
atopic dermatitis in both children and infants


39) A boy is noted at birth to have coarse scales over his trunk and extremities. The face, palms,
soles and flexures are spared. What is the least likely association?


A. Ectropion Correct Choice

B. Corneal opacities

C. Cryptorchidism

D. Neurologic abnormality

E. Prolonged maternal labor
X-linked ichthyosis is characterized by small, dark, firmly adherent scales accentuated on the sides
of the neck and trunk. The face, palms, soles, antecubital and popliteal flexures are generally
spared. Associated extracutaneous findings include corneal opacities (50%), undescended testes
(20%), and prolonged maternal labor (usual). Neurological or mental retardation are rare but
documented associations as XLI can be associated with a contiguous gene syndrome with Kallman
syndrome, mental retardation and X linked recessive chrondrodysplasia punctata




                                                  10
40) Which of the following is not a major criterion for Kawasaki’s disease:

A. Palmoplantar erythema > desquamation

B. Cardiac aneurysm Correct Choice

C. Fever >5 days

D. Strawberry tongue/ red lips

E. Cervical adenopathy
Cardiac aneurysm is a serious complication of Kawasaki’s disease. However, as the cardiovascular
manifestations generally present 1 –5 months after presentation, they are not criteria for diagnosis


41) Koplik spots typically appear:

A. At the same time as the exanthem

B. 1 week after the exanthem

C. 8 weeks after the exanthem

D. 6 weeks after the exanthem

E. Before the exanthem Correct Choice
The enanthem of measles precedes the morbilliform eruption.


42) Pastia’s lines are characteristic for which eruption:

A. Kawasaki’s disease

B. Scarlet fever Correct Choice

C. Rubella

D. Mumps

E. Measles
Pastia’s lines are defined as a linear accentuation of the erythematous sandpaper rash within the
flexures. This is classically described with scarlet fever


43) Which enanthem is most commonly seen in association with Exanthem subitum?

A. Koplik spots

B. Red strawberry tongue

C. Red macules and streaks on the soft palateCorrect Choice

D. Palatal erosions

E. Chapped lips; dry, red mucosa
Exanthem subitum (roseola or Sixth disease) is caused by HHV6, a dsDNA virus. It presents with
high fever for several days followed by an exanthem of erythematous macules and papules on the
trunk that begins as the fever ends. An associated enanthem of red macules/streaks on the soft
palate may be seen. Koplik spots are seen in measles; the red strawberry tongue (following the
white strawberry tongue) is seen in Scarlet fever; chapped lips and dry, red mucosa may be seen in
Kawasaki's disease; palatal erosions may be seen in Papular-purpuric gloves and socks syndrome




                                                  11
44) An 8 year-old boy presents with pink, flushed cheeks and a low-grade fever. On week later, the
following lacy eruption appeared. What is the most likely etiology?


A. Togavirus

B. HHV6

C. Group A streptococcus

D. Paramyxovirus

E. Parvovirus Correct Choice
Fifth disease is caused by Parvovirus B19. Most cases start with prodrome of fever, malaise,
headache and rhinorrhea. Cutaneous reaction follows approximately 5-7 days later with erythema of
the cheeks ("slapped cheeks") and reticulate rash of the trunk and extremities


45) An infant presents with red-purple, granulomatous nodules occurring in the diaper area. The
etiology is secondary to local irritation, maceration and Candida albicans. What is the most likely
diagnosis?


A. Langerhans cell histiocytosisc.

B. Biotin deficiency

C. Seborrheic dermatitis

D. Psoriasis

E. Granuloma gluteale infantum Correct Choice
The etiology of granuloma gluteale infantum is multifactorial, resulting from the unique environment
of the diaper area. Treatment consists of topical antifungal agents, barrier creams, and anti-
inflammatory agents as needed


46) The Carney complex is associated with a defect in:

A. LYST

B. PTEN

C. PRKAR1A Correct Choice

D. MASH2

E. MLH1
A PRKAR1A gene defect is associated with the Carney complex


47) What syndrome is the disorder shown in the photo associated with?

A. Noonan syndrome Correct Choice

B. Bloom syndrome

C. Turner syndrome

D. Down syndrome

E. Griscelli syndrome
The photo shows keratosis pilaris atrophicans faceii and surgically corrected ptosis. Both of these
findings are associated with Noonan syndrome




                                                  12
48) A 6 month-old presents with orange-brown crusted plaques around the mouth and groin.
Several bullae are present on the fingers and toes. Which of the following laboratory values is likely
to be abnormal?


A. Calcium

B. Hematocrit

C. Alkaline phosphatase Correct Choice

D. Platelet count

E. ALT
The most likely diagnosis is acrodermatitis enteropathica. Alkaline phosphatase is a zinc dependant
enzyme that is decreased in response to low serum zinc levels


49) Which of the following should be the next step in the management of this patient?

A. spine x-ray

B. Administration of oral antibiotics

C. Cardiac evaluationCorrect Choice

D. Barium swallow

E. Pulmonary function studies
This patient must be evaluated thoroughly for PHACES syndrome. PHACES is an acronym for
Posterior fossa malformations (Dandy-Walker malformation is most common), Hemangiomas,
Arterial anomalies, Coarctation of the aorta, Eye abnormalities, and Sternal cleft defects. This
patient should have a complete cardiac evaluation, neuroimaging, and ophthalmologic exam. If the
facial hemangioma involves the beard area, this may indicate laryngeal involvement and
appropriate imaging and evaluation is mandated. Systemic steroids at high doses (5 mg/kg) are
usually administered. If caught early, the sequelae of PHACES syndrome can be minimized


50) The association of Port-wine stains on a limb with soft tissue swelling with or without bony over
growth is:


A. Sturge-Weber syndrome

B. Goldenhar's syndrome

C. Klippel-Trenaunay syndromeCorrect Choice

D. Bannayan-Riley-Ruvalcaba syndrome

E. Proteous syndrome
The association of port-wine stain on a limb with soft tissue swelling with or without bony
overgrowth is Klippel-Trenuanay syndrome. Klippel-Trenaunay syndrome is characterized by the
Triad of port-wine malformations in association with deep venous system malformations, superficial
varicosities, and bony and soft tissue hypertrophy. Sturge-Weber syndrome has 2 essential
components: Facial port-wine stain and homolateral leptomeningeal angiiomattosis. The port wine
stain most commonly involves the areas innervated by the ophthalmic(V1) and maxillary (V2)
divisions of the trigeminal nerve. Complications of leptomeningeal angiomatosis are epilepsy,
mental retardation, and occasionally, contralateral hemiplegia. Proteus Syndrome is characterized
by vascular malformations including nevus flammeus, hemihypertrophy, macrodactyly, verrucous
epidermal nevus, soft-tissue subcutaneous masses, and cerebriform overgrowth of the plantar
surface. Babbyan-Riley-Ruvalcaba syndrome may include multiple cutaneous and visceral venous,
capillary, and lympathtic malformations, macroephaly, pseudopapilledema, systemic
lipoangiomatosis, spotted pigmentation of the penis, hamartomatous intestinal polyps, and rarely
trichilemmonmas. (multiple subcutaneous lipomas as well as acanthosis nigricans).



                                                  13
51) The most common age group for papular-purpuric gloves and socks syndrome is:

A. Newborns

B. 6-10 year olds

C. Toddlers

D. Adolescents Correct Choice

E. Elderly
This unique presentation of parvovirus infection typically occurs in adolescents and young adults


52) What is the most likely diagnosis?

A. Traction alopecia

B. Nevus sebaceous

C. Alopecia areata

D. Tinea capitis

E. Aplasia cutis congenita Correct Choice
Aplasia cutis congenita is characterized by the absence of a portion of skin, most commonly
presenting as a solitary defect on the scalp, but sometimes it may occur as multiple lesions. The
lesions are non-inflammatory and well demarcated appear as an atrophic, membranous, ulcerated
area with alopecia. The condition may be associated with other physical anomlies


53) POEMS syndrome is associated with which of the following:

A. M proteinCorrect Choice

B. Odontogenic cysts

C. Saddle nose deformity

D. Premature aging

E. Eye abnormalities
The acronym POEMS stands for polyneuropathy, organomegaly, endocrinopathy, M-protein, and
skin changes


54) Osteopathia striata is found in which disorder?

A. Gorlin's syndrome

B. Neurofibromatosis I

C. McCune-Albright syndrome

D. Buschke-Ollendorff syndrome

E. Focal dermal hypoplasiaCorrect Choice
Osteopathia striata (vertical striations in the metaphysis of long bones on x-ray) is seen is greater
than 80% of cases of focal dermal hypoplasia (or Goltz syndrome). Polyostotic fibrous dysplasia
with recurrent fractures is seen in McCune-Albright syndrome. Sphenoid wing dysplasia and thinning
of long bone cortex is found in neurofibromatosis I. Osteopoikilosis is an asymptomatic x-ray finding
in patients with Buschke-Ollendorf syndrome. Osteopoikilosis reflects ectopic calcification that does




                                                 14
not increase risk of fracture. Bifid ribs, vertebral fusion/Sprengel deformity of the spine, and
kyphoscoliosis can be seen in basal cell nevus syndrome (Gorlin's syndrome).


55) Which of the following is a ssDNA virus:

A. Parvovirus Correct Choice

B. Parapox

C. Herpesvirus

D. Picornovirus

E. Adenovirus
Parvovirus is the only ssDNA virus listed


56) Which treatment choice would be contraindicated in a one-year old child who presnents with
monomorphous, nonpruritic flat-topped papules on the face, buttocks, extremities, palms and soles?


A. Observation

B. Aspirin

C. CorticosteroidsCorrect Choice

D. Acetaminophen

E. Hydration
Gianotti-Crosti or papular acrodermatitis of childhood is associated with a variety of viral infections.
Patients have a typical cutaneous manifestation, low-grade fever, mild lymphadenopathy and
diarrhea. Corticosteroids should be avoidedas they may have an adverse effect


57) The standard of care of patients with acute Kawasaki’s disease is:

A. Supportive care

B. Aspirin and IVIGCorrect Choice

C. Penicillin

D. Acetaminophen and IVIG

E. Prednisone
Kawasaki disease, also called mucocutaneous lymph node syndrome, is an acute febrile disorder
based on the clinical criteria of changes in peripheral extremities, polymorphous exanthema,
conjunctival injection without exudates, changes in the lips or oral cavity, acute cervical
lymphadenopathy. Fever must be present, lasting more than 5 days. Treatment is aimed to prevent
coronary aneurysms and myocardial infarction. Treatment for acute Kawasaki disease is intravenous
immunoglobulin 2 g/kg over 10-12 hours and aspirin therapy


58) A newborn has a nodule over his lumbar spine. Skin biopsy reveals a lipoma. The next
appropriate step is:


A. Genetic testing

B. Observation

C. Imaging studyCorrect Choice

D. Malignancy work up




                                                   15
E. Excision of the lesion
The skin can provide an important clue to the presence of an underlying neural tube defect, such as
meningomyelocele and encephalocele. Cutaneous lesions along the midline of the spine should
always prompt consideration of this possibility. Although, midline neural tube defects are
uncommon, early recognition and diagnosis of a spinal dysraphism can have important implications
for early surgical correction and minimizing loss of neurologic function. Clues to the diagnosis
include a midline dimple, tuft of hair, lipoma, or vascular lesion. In these instances, imaging studies
(MRI, CT, ultrasound) should be promptly initiated


59) An infant presents with the lesion depicted in the photo. Which of the following is least likely?

A. Dandy-Walker malformation

B. Congenital cataracts

C. Sternal clefting

D. Supraumbilical raphe

E. Seizure disorder Correct Choice
Extensive facial hemangiomas are a component of the PHACES syndrome. Seizure disorder has not
been described


60) A 24 month-old infant presents with yellowish-brown, crusted papules with petechiae in a
seborrheic distribution. A biopsy is done to confirm a diagnosis. Which histologic picture is most
likely?


A. CD1-, S100- cells with reniform nuclei

B. Mixed cellular infiltrate in a “ball and claw” pattern

C. Foamy histiocytes with Touton giant cells

D. Superficial perivascular infiltrate with mild spongiosis and neutrophil containing scale crust

E. CD1+, S100+ cells with reniform nuclei Correct Choice
Langerhans cells are CD1 and S100 positive. The nuclei are described as kidney shaped, or reniform


61) What is the function of the gene which is defective in ataxia-telangiectasia?

A. Pathway of cholesterol biosynthesis

B. Cross-linking of structural proteins in the protein and lipid envelope of the upper epidermis

C. Gap junction protein

D. DNA repair proteinCorrect Choice

E. Tumor supressor protein
The defective gene is ataxia-telangiectasia (Louis-Bar syndrome) is the ATM gene, which is
responsible for DNA repair, especially after ionizing radiation. Tumor suppressor genes mutations
are responsible for basal cell nevus syndrome, xeroderma pigmentosum, Muir-Torre syndrome,
dyskeratosis congenital, Gardner syndrome, Peutz-Jeghers syndrome, Cowden syndrome, and MEN
syndromes. Connexins are gap junction proteins that are responsible for intercellular communication
and signaling. Mutations in connexins are responsible for Vohwinkel syndrome and
erythrokeratoderma variabilis. Mutations in the cholesterol biosynthesis pathways cause CHILD
syndrome and Conradi-Hunermann syndrome. Tranglutaminase 1 (TGM 1) is involved in the normal
cross-linking of structural proteins in the protein and lipid envelope of the upper epidermis. TGM 1 is
mutated in lamellar ichthyosis and congenital ichthyosiform erythroderma




                                                    16
62) What is the diagnosis?

A. Rubella

B. Contact dermatitis

C. Unilateral laterothoracic exanthem Correct Choice

D. Urticaria

E. Roseola
The child in the photo depicts the “Statue of Liberty” sign of unilateral laterothoracic exanthem


63) The most appropriate treatment of a 2 month-old with a hemangioma involving the lateral
canthus is:


A. Observation and reassurance of the parents

B. Pulsed dye laser

C. Surgical excision

D. Interferon-alpha

E. Systemic corticosteroidsCorrect Choice
Since hemangiomas characteristically follow a course of proliferation followed by spontaneous
involution, many of the lesions can be followed with conservative management including
observation. However, indications for treatment of hemangiomas include obstruction of vital
function, high-output cardiac failure, ulceration, infection, diaper area location and location on the
face. Aggressive treatment is indicated for a hemangioma located on the lateral canthus since two
of the above indications apply. Importantly, obstruction of the visual field may impair development
of the visual cortex in an infant. Oral glucocorticoids are the mainstay of treatment for
hemangiomas, with 30 to 60% of lesions responsive to therapy. Alternative treatments for
problematic hemangiomas include intralesional steroids, interferon alpha-2a, and pulsed dye laser


64) Neonatal acne is associated with species of which organism:

A. Streptococcus

B. Candida

C. MalasseziaCorrect Choice

D. Staphylococcus

E. Propionibacterium
Acne which develops within the first 30 days of life is termed neonatal acne. Neonatal acne has a
predilection for the face, chest, back and groin appearing as small, discrete papules at 2 to 4 weeks
of age, and persisting for up to 8 months. As these lesions are self-resolving, no treatment is
necessary, though 2.5% benzoyl peroxide may hasten resolution. Neonatal acne is quite common
and is postulated to occur as a result of hyperplasia of premature sebaceous glands coupled with
transient increases in circulating androgens. More recent data suggests that Malassezia species may
be implicated at etiologic factors in neonatal acne. These organisms have been cultured from the
skin of affected patients, though their exact role in unclear


65) What is the most likely diagnosis?

A. Cutaneous T-cell lymphoma

B. Psoriasis




                                                  17
C. Contact dermatitisCorrect Choice

D. Lichen planus

E. Pityriasis rosea
This periumbilical eruption is classic for a contact nickel allergy. The metal snaps on pants are the
cause in this case


66) What is the best therapeutic option?

A. Oral cephalexin

B. Oral prednisone

C. Oral acyclovir Correct Choice

D. Topical mupirocin

E. Topical immune modulator
The condition shown is eczema herpeticum. The most appropriate treatment is oral acyclovir


67) The most likely etiology of Jacquet’s diaper dermatitis is:

A. Multifactorial Correct Choice

B. Candida

C. Trichophyton rubrum

D. Herpes simplex virus, Type 2

E. Group A beta-hemolytic streptococcus
Jacquet’s diaper dermatitis is a multifactorial process. Yeast, irritants and moisture all contribute to
the occurrence of this eruption


68) The causative agent of Roseola is:

A. A dsRNA virus

B. A ssDNA virus

C. Streptococcus

D. A dsDNA virus Correct Choice

E. A ssRNA virus
Roseola is caused by Human Herpesvirus 6, a double stranded DNA virus


69) Which of the following is the most common complication associated with cutis marmorata
telangectatica congenita?


A. Systemic lupus erythematosus

B. Hypercalcemia

C. Limb hypertrophy or atrophy Correct Choice

D. Atrial septal defect

E. Seizure disorder




                                                   18
Hypertrophy or atrophy of the affected limb is the most likely consequence of cutis marmorata
telangectatica congenita. Orthopedic evaluation should be a part of the patient’s routine
management


70) Which of the following may be associated?

A. Atrial septal defect

B. AVM

C. Paronychia

D. Seizure disorder Correct Choice

E. Cleft palate
Nevus sebaceus can very rarely be associated with multiple anomalies. Schimmelpenning syndrome
can include seizure disorder, mental retardation, coloboma, as well as skeletal, cardiac and
genitourinary abnormalities


71) A 4 month-old with difuse blisters and erosions has a skin biopsy diagnostic of generalized
mastocytosis. Which topical dressing should be avoided in this patient?


A. Petrolatum

B. Mupirocin ointment

C. Polymyxin B ointmentCorrect Choice

D. Neomycin ointment

E. Silver sulfadiazine
Mastocytosis compromises a group of diseases characterized by increased number of mast cells in
the skin and other organs. Seventy-five percent of cases occur before the age of 2. Patients with
mastocytosis should avoid potential mast degranulators including aspirin, codeine, opiates,
procaine, spicy foods, cheese, alcohol, polymyxin B.


72) A neonate presents with a large segmental hemangioma of the V1 distribution. What are
features may be associated with this finding?


A. Posterior fossa defectCorrect Choice

B. Tram-track calcifications

C. Muscular dystrophy

D. M-paraproteinemia

E. Anterior fossa defect
PHACES syndrome consists of posterior fossa defect, hemangiomas (often segmental and large),
arterial defects, cardiac defects, eye abnormaliteis, and sternal clefting. Posterior fossa defects
include the Dandy-Walker malformtion


73) Late onset subungual keratotic tumors are associated with:

A. Basal cell nevus syndrome

B. Incontinentia pigmenti Correct Choice

C. Neurofibromatosis Type 1




                                                  19
D. Cowden syndrome

E. Carney complex
A NEMO gene defect can cause subungual keratotic growths. The typical age of presentation is early
adulthood


74) A 2 week-old infant is brought to the ER with a rash on her face. She is found to have a 3rd
degree heart block. What is the risk that a second child born to this mother will have the same
diagnosis?


A. 100%

B. 10%

C. 25%Correct Choice

D. 5%

E. 50%
The diagnosis here is neonatal lupus erythematosus. Babies are normal at birth and develop skin
lesions within the few months of life. About half of these babies will have an associated congenital
heart block, usually 3rd degree, which is permanent. Most infants with NLE are girls and are born to
mothers who are Ro/La positive. There is a 25% chance that a second child with have NLE


75) A newborn presents with a well-defined, shiny patch with complete alopecia on the vertex of
the scalp along the suture lines. Which of the following is the first step in diagnosis?


A. Calcium

B. Skin biopsy

C. Fungal culture

D. MRI

E. Skull x-ray Correct Choice
The newborn most likely has aplasia cutis congenita. A skull x-ray would be the simplest, most cost
effective means of identifying any underlying bony abnormality


76) Multiple lesions are associated with a defect in:

A. Patched

B. Ras

C. Beta-catenin Correct Choice

D. NEMO

E. PTEN
The bluish cystic nodule on the face of this boy is a pilomatrichoma. A defect in beta-catenin has
been implicated


77) Which of the following disorders is more likely to occur in children with chronic fecal
incontinence?


A. Seborrheic dermatitis

B. Perianal streptococcal disease




                                                   20
C. Perianal pseudoverrucous papules and nodules Correct Choice

D. Granuloma gluteale infantum

E. Langerhans cell histiocytosis
The warty papules of perianal pseudoverrucous papules and nodules are seen more commonly in
the setting of chronic fecal incontinence


78) All four subtypes of Phakomatosis Pigmentovascularis have which feature in common:

A. Nevus spilus

B. Nevus anemicus

C. Epidermal nevi

D. Nevus flammeusCorrect Choice

E. Dermal Melanocytosis.
Phakomatosis Pigmentovascularis: Patients with a combination of vascular malformations and
melanocytic or epidermal nevi are grouped into 4 subtypes of this disorder. All have nevus
flammeus/capillary malformation (CM).

Type   I: CM + epidermal nevus
Type   II: CM + dermal melanocytosis +/- nevus anemicus
Type   III: CM + nevus spilus +/- nevus anemicus
Type   IV: CM + dermal melanocytosis + nevus spilus +/- nevus anemicus


79) The disorder caused by a defect in intestinal zinc-specific transporter SLC39A4 is most likely to
present:


A. At birth

B. In adulthood

C. Upon weaning from breast milk Correct Choice

D. In childhood

E. Upon weaning from formula
In acrodermatitis enteropathica, there is decreased absorption of zinc from the infant’s
gastrointestinal tract. The zinc in breast milk has greater availability than nonmaternal sources thus
protecting the child from disease expression until weaning


80) The lesion depicted is most often associated with:

A. Polyostotic fibrous dysplasia Correct Choice

B. Chondrodysplasia punctata

C. Cleft palate

D. Osteopoikilosis

E. Sphenoid wing hypoplasia
The “coast of Maine” café au lait macule is associated with McCune-Albright syndrome and
polyostotic fibrous dysplasia


81) What is the most likely diagnosis?



                                                  21
A. German measles

B. Letterer-Siwe disease

C. Mucocutaneous lymph node syndrome

D. Exanthem subitum

E. Papular acrodermatitis of childhoodCorrect Choice
Gianotti-Crosti syndrome is also known as papular acrodermatitis of childhood. The eruption is
characterized by lichenoid papules in an acral distribution


82) A full term neonate is noted to have small pustules with no underlying erythema present at
delivery. The pustules are easily removed with clearing of the vernix and a collarette appears. A
gram stain is done showing predominately neutrophils without bacteria. What is the most likely
diagnosis?


A. Miliaria

B. Transient neonatal pustular melanosis Correct Choice

C. Urticaria pigmentosa

D. Erythema toxicum neonatorum

E. Congenital candidiasis
Transient neonatal pustular melanosis typically begins with sterile pustules that leave a
characteristic collarette when ruptured. The lesions heal with hyperpigmented macules


83) What is the most likely diagnosis:

A. Facticial dermatosis

B. Epidermolysis bullosa simplex Correct Choice

C. Linear bullous IgA disease

D. Epidermolysis bullosa dystrophica

E. Pemphigus vulgaris
The bullous lesions represented are nonscarring and are associated with obvious nail dystrophy.
Epidermolysis bullosa simplex, Dowling-Meara type, is the best option


84) Which of the following diseases with immunodeficiency has an increased risk of lymphoreticular
malignancy?


A. Chronic granulomatous disease

B. Severe combined immunodeficiency syndrome

C. Leiner’s disease

D. Wiskott-Aldrich syndromeCorrect Choice

E. Job syndrome
Wiskott-Aldrich syndrome is an X-linked recessive disorder caused by mutations in WAS gene.
Patients with Wiskott-Aldrich have atopic dermatitis with increased risks for secondary infection,
thrombocytopenia, and recurrent bacterial infections. They are also at an increased risk for
lymphoreticular malignancy (20%




                                                  22
85) Which of the following is the most common long term sequelae from congenital rubella
syndrome?


A. Nystagmus

B. Macrocephaly

C. Microcephaly

D. Deafness Correct Choice

E. Saber shins
Deafness may occur in up to 50% of infants with congenital rubella syndrome


86) Rhinorrhea, condylomata lata, and mucous patches are all seen with which congenital disorder?

A. Human papillomavirus infection

B. Herpes simplex virus

C. Rubella

D. Toxoplasmosis

E. Syphilis Correct Choice
Signs of congenital syphilis include rhinorrhea, snuffles, rhagades, condylomata lata, and mucous
patches. Condylomata lata or a generalized papulosquamous eruption of secondary syphilis may be
present in diaper area


87) What is the most likely diagnosis?

A. Langerhans cell histiocytosis

B. Perianal streptococcal disease Correct Choice

C. Granuloma gluteale infantum

D. Psoriasis

E. Contact dermatitis
The bright red erythema of perianal streptococcal disease can also involve the creases of the groin
and axillae


88) What is the most likely diagnosis?

A. Bullous pemphigoid of infancy

B. Herpes zoster

C. Epidermolysis bullosa simplex

D. Incontinentia pigmenti Correct Choice

E. Disseminated herpes simplex
The vesicular lesions following the lines of Blaschko are typical for incontinentia pigmenti


89) Which of the following is most likely associated?

A. No additional abnormality Correct Choice




                                                   23
B. Alopecia

C. Deafness

D. Bifid rib

E. Coarctation of the aorta
The majority of patients with accessory tragii do not have any associated abnormalities. Multiple
anomalies, including deafness and midline defects, have been reported but are rare


90) What is the most likely diagnosis?

A. Seborrheic dermatitis

B. Langerhans cell histiocytosis

C. Psoriasis Correct Choice

D. Perianal streptococcal disease

E. Contact dermatitis
The well-defined erythematous plaque is an example of psorisis in the diaper area. The moist nature
of the diaper environment results in a loss of the classic silvery scale


91) A healthy, full-term infant develops a pustular, erythematous eruption on her face and trunk on
the third day of life. A smear taken from one of these pustules would show:


A. Predominantly neutrophils

B. Predominantly eosinophilsCorrect Choice

C. Gram-positive bacteria

D. Hyphae

E. Multi-nucleated giant cells
This baby has developed erythema toxicum neonatorum, a benign, self-limited eruption that occurs
in the majority of healthy, full-term infants. Erythema toxicum usually develops on the second or
third day of life and resolves by day 10. A smear taken from one of the pustules will demonstrate
eosinophils, which is sufficient to make the diagnosis. Viral infections of the skin may demonstrate
multi-nucleated giant cells on Tzanck smear. Transient neonatal pustular melanosis is present at
birth; neonates present with small, superficial pustules that rupture easily. Some may have
ruptured in utero, leaving pigmented macules. The pigmentation may persist for weeks to months in
darkly-complexed infants. A smear of the intracorneal/subcorneal pustule will show mostly
neutrophils, but eosinophils may also be present


92) Which of the following clinical signs is most likely associated?

A. Alopecia

B. Hypertrichosis

C. Clitoral hypertrophy Correct Choice

D. Neurofibromas

E. Axillary freckling
The “coast of Maine” café au lait macule is associated with McCune-Albright syndrome. Endocrine
abnormalities, including precocious puberty and polyostotic fibrous dysplasia are associated
features. The genetic mutation which results in this genodermatosis is due to a post-somatic
mutaion in the alpha sub-unit of stimulatory G-protein



                                                  24
93) A newborn infant presents with ring of long, dark, coarse hair surrounding a midline scalp patch
of alopecia. What is associated with this finding?


A. Thyroid disease

B. Alopecia areata

C. Ectopic brain tissueCorrect Choice

D. Deafness

E. Nevus sebaceus of Jadassohn
The "hair collar sign" is associated with ectopic brain tissue and is thought to arise from a congenital
herniation through the skull. Caution must be used in evaluation as biopsy or needle aspiration may
lead to retrograde infection


94) What deficiency is responsible for this condition?

A. Vitamin A

B. Thiamine

C. Vitamin C

D. ZincCorrect Choice

E. Niacin
Acrodermatitis enteropathica is a rare inherited disorder transmitted in an autosomal recessive
fashion. The disorder is caused by an inability to absorb zinc. the clinical syndrome is characterized
by acral dermatitis, alopecia, and diarrhea. Zinc deficiency from other causes (including chronic
wasting, poor oral intake, lack of supplementation in total parenteral nutrition) can cause similar
clinical changes


95) The genetic disorder depicted in this photo is caused by a mutation in:

A. STK11 Correct Choice

B. Patched

C. TSC1

D. PTEN

E. MASH2
The multiple labial lentigo seen on this young boy are seen with Peutz-Jegher syndrome. A defect in
the STK11 gene is responsible


96) A 2 year-old has a high fever for three days, as the fever breaks, a generalized rash appears on
the trunk. The most likely diagnosis is:


A. Rubella

B. Erythema infectiosum

C. Measles

D. Roseola Correct Choice

E. Mumps




                                                     25
Roseola typically appears in toddlers. A high fever followed by a generalized maculopapular rash is
characteristic for this HHV6 infection


97) What is the most likely neoplasm associated with this lesion?

A. Keratoacanthoma

B. Syringocystadenoma papilliferumCorrect Choice

C. Clear cell acanthoma

D. Basal cell carcinoma

E. Pilomatricoma
Nevus sebaceus is usually a solitary lesion that presents at birth or shortly after. It usually has a
characteristic yellow or yellow brown color with a verrucous surface. Syringocystadenoma
papilliferum is the most common neoplasm associated with nevus sebaceous


98) What is the most likely diagnosis is this 16 year old patient who developed generalized eruption
2 weeks after onset of sore throat due to strep infection?


A. Lichen planus

B. Cutaneous T-cell lymphoma

C. Pityriasis rosea

D. Psoriasis Correct Choice

E. Contact dermatitis
Although the exact mechanism is not known, there is a well-known correlation between a history of
upper respiratory infection secondary to group A beta-hemolytic streptococci and the subsequent
development of guttate psoriasis. The eruption may resolve spontaneously or with treatment
(topical steroids or ultraviolet therapy). Some portion of these patients eventually develop chronic,
plaque-type psoriasis


99) Psammomatous melanotic schwannomas are associated with:

A. Tuberous sclerosis

B. Neurofibromatosis Type 2

C. Bloom syndrome

D. Carney complex Correct Choice

E. Neurofibromatosis Type 1
Psammomatous melanotic schwannomas have been described in the Carney complex, a defect in
the tumor suppressor gene, PRKAR1A


100) Rapp-Hodgkin is caused by a defect in the following gene:

A. Ectodysplasin A

B. P63Correct Choice

C. Desmoglein 1

D. Plakophilin




                                                   26
E. Connexin 30
Rapp-Hodgkin, along with the other ectodermal dysplasia-clefting syndromes, is reportedly caused
by a defect in the p63 gene


101) The differential diagnosis of zinc deficiency is least likely to include:

A. Cystic fibrosis

B. Holocarboxylase synthetase deficiency

C. Biotin deficiency

D. Granuloma gluteale infantum Correct Choice

E. Multiple carboxylase deficiency
All of the options result in an eczematous acrodermatitis enteropathica-like eruption except
granuloma gluteale infantum. As the name suggests, the lesions of granuloma gluteale infantum are
granulomatous




                                                   27

Más contenido relacionado

La actualidad más candente

ETAS_MCQ_12 bullous diseases and blistering
ETAS_MCQ_12 bullous diseases and blisteringETAS_MCQ_12 bullous diseases and blistering
ETAS_MCQ_12 bullous diseases and blisteringDerma202
 
ETAS_MCQ_03 a genodermatoses
ETAS_MCQ_03 a genodermatosesETAS_MCQ_03 a genodermatoses
ETAS_MCQ_03 a genodermatosesDerma202
 
ETAS_MCQ_15 dermatologic and cosmetic surgery
ETAS_MCQ_15 dermatologic and cosmetic surgeryETAS_MCQ_15 dermatologic and cosmetic surgery
ETAS_MCQ_15 dermatologic and cosmetic surgeryDerma202
 
Arab board primary exam in dermatology 2012
Arab board primary exam  in dermatology 2012Arab board primary exam  in dermatology 2012
Arab board primary exam in dermatology 2012Derma202
 
Dermatology board review
Dermatology board reviewDermatology board review
Dermatology board reviewAhmed Amer
 
Dermatology approach
Dermatology approachDermatology approach
Dermatology approachFayzaRayes
 
ETAS_MCQ_05 dermatopathology
ETAS_MCQ_05 dermatopathologyETAS_MCQ_05 dermatopathology
ETAS_MCQ_05 dermatopathologyDerma202
 
Eczema and dermatitis DERMATOLOGY
Eczema and dermatitis DERMATOLOGYEczema and dermatitis DERMATOLOGY
Eczema and dermatitis DERMATOLOGYTONY SCARIA
 
Pediatric Dermatology Spot Diagnosis
Pediatric Dermatology Spot DiagnosisPediatric Dermatology Spot Diagnosis
Pediatric Dermatology Spot DiagnosisFatima Farid
 
Krok 1 - 2015 (Pharmacology)
Krok 1 - 2015 (Pharmacology)Krok 1 - 2015 (Pharmacology)
Krok 1 - 2015 (Pharmacology)Eneutron
 
Dermatolody quizzes
Dermatolody quizzesDermatolody quizzes
Dermatolody quizzesFayzaRayes
 
Class iii english
Class iii englishClass iii english
Class iii englishRc Os
 
Soal SKI Kelas VII Semester 1
Soal SKI Kelas VII Semester 1Soal SKI Kelas VII Semester 1
Soal SKI Kelas VII Semester 1Baitinnajmah
 

La actualidad más candente (20)

ETAS_MCQ_12 bullous diseases and blistering
ETAS_MCQ_12 bullous diseases and blisteringETAS_MCQ_12 bullous diseases and blistering
ETAS_MCQ_12 bullous diseases and blistering
 
ETAS_MCQ_03 a genodermatoses
ETAS_MCQ_03 a genodermatosesETAS_MCQ_03 a genodermatoses
ETAS_MCQ_03 a genodermatoses
 
ETAS_MCQ_15 dermatologic and cosmetic surgery
ETAS_MCQ_15 dermatologic and cosmetic surgeryETAS_MCQ_15 dermatologic and cosmetic surgery
ETAS_MCQ_15 dermatologic and cosmetic surgery
 
Arab board primary exam in dermatology 2012
Arab board primary exam  in dermatology 2012Arab board primary exam  in dermatology 2012
Arab board primary exam in dermatology 2012
 
Dermatology for MRCP
Dermatology for MRCPDermatology for MRCP
Dermatology for MRCP
 
Dermatology board review
Dermatology board reviewDermatology board review
Dermatology board review
 
Dermatology approach
Dermatology approachDermatology approach
Dermatology approach
 
ETAS_MCQ_05 dermatopathology
ETAS_MCQ_05 dermatopathologyETAS_MCQ_05 dermatopathology
ETAS_MCQ_05 dermatopathology
 
Icthyosis
IcthyosisIcthyosis
Icthyosis
 
Eczema and dermatitis DERMATOLOGY
Eczema and dermatitis DERMATOLOGYEczema and dermatitis DERMATOLOGY
Eczema and dermatitis DERMATOLOGY
 
1.1.1. bacterial infection of skin [compatibility mode]
1.1.1. bacterial infection of skin [compatibility mode]1.1.1. bacterial infection of skin [compatibility mode]
1.1.1. bacterial infection of skin [compatibility mode]
 
Pediatric Dermatology Spot Diagnosis
Pediatric Dermatology Spot DiagnosisPediatric Dermatology Spot Diagnosis
Pediatric Dermatology Spot Diagnosis
 
Mrcp 2 dermatology
Mrcp 2 dermatologyMrcp 2 dermatology
Mrcp 2 dermatology
 
Bullous skin diseases
Bullous skin diseasesBullous skin diseases
Bullous skin diseases
 
Krok 1 - 2015 (Pharmacology)
Krok 1 - 2015 (Pharmacology)Krok 1 - 2015 (Pharmacology)
Krok 1 - 2015 (Pharmacology)
 
Dermatolody quizzes
Dermatolody quizzesDermatolody quizzes
Dermatolody quizzes
 
Class iii english
Class iii englishClass iii english
Class iii english
 
Disorders of pigmentation
Disorders of pigmentationDisorders of pigmentation
Disorders of pigmentation
 
Soal SKI Kelas VII Semester 1
Soal SKI Kelas VII Semester 1Soal SKI Kelas VII Semester 1
Soal SKI Kelas VII Semester 1
 
Dermoscopy an overview
Dermoscopy  an overviewDermoscopy  an overview
Dermoscopy an overview
 

Destacado

Dermatology without pics
Dermatology without picsDermatology without pics
Dermatology without picsess_online
 
Psoriasis - A BRIEF OUTLOOK..................................by Vishnu R.Nair...
Psoriasis - A BRIEF OUTLOOK..................................by Vishnu R.Nair...Psoriasis - A BRIEF OUTLOOK..................................by Vishnu R.Nair...
Psoriasis - A BRIEF OUTLOOK..................................by Vishnu R.Nair...RxVichuZ
 
ETAS_MCQ_10 manifestations of systemic diseases1
ETAS_MCQ_10 manifestations of systemic diseases1ETAS_MCQ_10 manifestations of systemic diseases1
ETAS_MCQ_10 manifestations of systemic diseases1Derma202
 
ETAS_MCQ_11 disorder of hair and nails
ETAS_MCQ_11 disorder of hair and nailsETAS_MCQ_11 disorder of hair and nails
ETAS_MCQ_11 disorder of hair and nailsDerma202
 
Granulomatous diseases of nose
Granulomatous diseases of noseGranulomatous diseases of nose
Granulomatous diseases of noseRITURAJANMBBS
 
Cystic masses of neck
Cystic masses of neckCystic masses of neck
Cystic masses of neckPRAMODG11
 
Phototherapy treatment protocol
Phototherapy treatment protocolPhototherapy treatment protocol
Phototherapy treatment protocolDerma202
 
Adnexal tumours of the skin and familial syndromes.
Adnexal tumours of the skin and familial syndromes.Adnexal tumours of the skin and familial syndromes.
Adnexal tumours of the skin and familial syndromes.namrathrs87
 
Approach to the child with rash
Approach to the child with rashApproach to the child with rash
Approach to the child with rashAjit Gadekar
 

Destacado (12)

Dermatology without pics
Dermatology without picsDermatology without pics
Dermatology without pics
 
Pediatric dermatology
Pediatric dermatology Pediatric dermatology
Pediatric dermatology
 
Psoriasis - A BRIEF OUTLOOK..................................by Vishnu R.Nair...
Psoriasis - A BRIEF OUTLOOK..................................by Vishnu R.Nair...Psoriasis - A BRIEF OUTLOOK..................................by Vishnu R.Nair...
Psoriasis - A BRIEF OUTLOOK..................................by Vishnu R.Nair...
 
Psoriasis
PsoriasisPsoriasis
Psoriasis
 
ETAS_MCQ_10 manifestations of systemic diseases1
ETAS_MCQ_10 manifestations of systemic diseases1ETAS_MCQ_10 manifestations of systemic diseases1
ETAS_MCQ_10 manifestations of systemic diseases1
 
ETAS_MCQ_11 disorder of hair and nails
ETAS_MCQ_11 disorder of hair and nailsETAS_MCQ_11 disorder of hair and nails
ETAS_MCQ_11 disorder of hair and nails
 
Granulomatous diseases of nose
Granulomatous diseases of noseGranulomatous diseases of nose
Granulomatous diseases of nose
 
Cystic masses of neck
Cystic masses of neckCystic masses of neck
Cystic masses of neck
 
Phototherapy treatment protocol
Phototherapy treatment protocolPhototherapy treatment protocol
Phototherapy treatment protocol
 
Adnexal tumours of the skin and familial syndromes.
Adnexal tumours of the skin and familial syndromes.Adnexal tumours of the skin and familial syndromes.
Adnexal tumours of the skin and familial syndromes.
 
Approach to the child with rash
Approach to the child with rashApproach to the child with rash
Approach to the child with rash
 
Pediatric Dermatology
Pediatric DermatologyPediatric Dermatology
Pediatric Dermatology
 

Similar a ETAS_MCQ_09 pediatric dermatology

100 mc qsintropicalmedicine
100 mc qsintropicalmedicine100 mc qsintropicalmedicine
100 mc qsintropicalmedicineElyas Mohammed
 
Krok 1 Medicine - 2016 General Medicine
Krok 1 Medicine - 2016 General MedicineKrok 1 Medicine - 2016 General Medicine
Krok 1 Medicine - 2016 General MedicineEneutron
 
Internal Medicine Image Challenge MCQs
Internal Medicine Image Challenge MCQsInternal Medicine Image Challenge MCQs
Internal Medicine Image Challenge MCQsSherif Elbadrawy
 
100-Mcqs-In-Neonatology.ppt
100-Mcqs-In-Neonatology.ppt100-Mcqs-In-Neonatology.ppt
100-Mcqs-In-Neonatology.ppthussainAltaher
 
Deramatology MRCGP Qs
Deramatology MRCGP QsDeramatology MRCGP Qs
Deramatology MRCGP Qsssnsharifa
 
Neonatologymcqs 100403011632-phpapp02
Neonatologymcqs 100403011632-phpapp02Neonatologymcqs 100403011632-phpapp02
Neonatologymcqs 100403011632-phpapp02AmarSETIA
 
Krok 1 - 2008 Question Paper (General medicine)
Krok 1 - 2008 Question Paper (General medicine)Krok 1 - 2008 Question Paper (General medicine)
Krok 1 - 2008 Question Paper (General medicine)Eneutron
 
Krok 1 - 2006 Question Paper (General medicine)
Krok 1 - 2006 Question Paper (General medicine)Krok 1 - 2006 Question Paper (General medicine)
Krok 1 - 2006 Question Paper (General medicine)Eneutron
 
STEP 2 - Nums past paper by dr. shahid alam
STEP 2 - Nums past paper by dr. shahid alamSTEP 2 - Nums past paper by dr. shahid alam
STEP 2 - Nums past paper by dr. shahid alamDr. Shadab Kamal
 
Ganyang MCQ Infectious dss
Ganyang MCQ Infectious dssGanyang MCQ Infectious dss
Ganyang MCQ Infectious dssKirie Kozanegawa
 
Krok 2 - 2013 (Pediatrics)
Krok 2 - 2013 (Pediatrics)Krok 2 - 2013 (Pediatrics)
Krok 2 - 2013 (Pediatrics)Eneutron
 
A Beginners Guide To COMLEX Level 1 - (COMQUEST)
A Beginners Guide To COMLEX Level 1 - (COMQUEST)A Beginners Guide To COMLEX Level 1 - (COMQUEST)
A Beginners Guide To COMLEX Level 1 - (COMQUEST)COMQUESTOsteopathic
 
Krok 1 - 2014 Question Paper (General medicine)
Krok 1 - 2014 Question Paper (General medicine)Krok 1 - 2014 Question Paper (General medicine)
Krok 1 - 2014 Question Paper (General medicine)Eneutron
 

Similar a ETAS_MCQ_09 pediatric dermatology (20)

100 mc qsintropicalmedicine
100 mc qsintropicalmedicine100 mc qsintropicalmedicine
100 mc qsintropicalmedicine
 
Krok 1 Medicine - 2016 General Medicine
Krok 1 Medicine - 2016 General MedicineKrok 1 Medicine - 2016 General Medicine
Krok 1 Medicine - 2016 General Medicine
 
Internal Medicine Image Challenge MCQs
Internal Medicine Image Challenge MCQsInternal Medicine Image Challenge MCQs
Internal Medicine Image Challenge MCQs
 
100-Mcqs-In-Neonatology.ppt
100-Mcqs-In-Neonatology.ppt100-Mcqs-In-Neonatology.ppt
100-Mcqs-In-Neonatology.ppt
 
Newborn mcq
Newborn mcqNewborn mcq
Newborn mcq
 
Newborn mcq
Newborn mcqNewborn mcq
Newborn mcq
 
Deramatology MRCGP Qs
Deramatology MRCGP QsDeramatology MRCGP Qs
Deramatology MRCGP Qs
 
Neonatologymcqs 100403011632-phpapp02
Neonatologymcqs 100403011632-phpapp02Neonatologymcqs 100403011632-phpapp02
Neonatologymcqs 100403011632-phpapp02
 
Krok 1 - 2008 Question Paper (General medicine)
Krok 1 - 2008 Question Paper (General medicine)Krok 1 - 2008 Question Paper (General medicine)
Krok 1 - 2008 Question Paper (General medicine)
 
Gd oor 912
Gd oor 912Gd oor 912
Gd oor 912
 
Neck lumps
Neck lumpsNeck lumps
Neck lumps
 
Mcq ped neuro
Mcq ped neuroMcq ped neuro
Mcq ped neuro
 
Krok 1 - 2006 Question Paper (General medicine)
Krok 1 - 2006 Question Paper (General medicine)Krok 1 - 2006 Question Paper (General medicine)
Krok 1 - 2006 Question Paper (General medicine)
 
STEP 2 - Nums past paper by dr. shahid alam
STEP 2 - Nums past paper by dr. shahid alamSTEP 2 - Nums past paper by dr. shahid alam
STEP 2 - Nums past paper by dr. shahid alam
 
Ganyang MCQ Infectious dss
Ganyang MCQ Infectious dssGanyang MCQ Infectious dss
Ganyang MCQ Infectious dss
 
Mcqs for Ophthal pgs 1
Mcqs  for Ophthal pgs 1Mcqs  for Ophthal pgs 1
Mcqs for Ophthal pgs 1
 
Krok 2 - 2013 (Pediatrics)
Krok 2 - 2013 (Pediatrics)Krok 2 - 2013 (Pediatrics)
Krok 2 - 2013 (Pediatrics)
 
A Beginners Guide To COMLEX Level 1 - (COMQUEST)
A Beginners Guide To COMLEX Level 1 - (COMQUEST)A Beginners Guide To COMLEX Level 1 - (COMQUEST)
A Beginners Guide To COMLEX Level 1 - (COMQUEST)
 
Krok 1 - 2014 Question Paper (General medicine)
Krok 1 - 2014 Question Paper (General medicine)Krok 1 - 2014 Question Paper (General medicine)
Krok 1 - 2014 Question Paper (General medicine)
 
2006
20062006
2006
 

Más de Derma202

Histopathplogical photos
Histopathplogical photosHistopathplogical photos
Histopathplogical photosDerma202
 
Slide study from ETAS
Slide  study from ETASSlide  study from ETAS
Slide study from ETASDerma202
 
Dermatology
DermatologyDermatology
DermatologyDerma202
 
ETAS_MCQ_14 plants and creatures of dermatologic significance
ETAS_MCQ_14 plants and creatures of dermatologic significanceETAS_MCQ_14 plants and creatures of dermatologic significance
ETAS_MCQ_14 plants and creatures of dermatologic significanceDerma202
 
ETAS_MCQ_13 photobiology and photosensitivity disorders
ETAS_MCQ_13 photobiology and photosensitivity disordersETAS_MCQ_13 photobiology and photosensitivity disorders
ETAS_MCQ_13 photobiology and photosensitivity disordersDerma202
 
ETAS_MCQ_03 b genodermatoses
ETAS_MCQ_03 b genodermatosesETAS_MCQ_03 b genodermatoses
ETAS_MCQ_03 b genodermatosesDerma202
 
Derm handbook for medical students and junior doctors 2010
Derm handbook for medical students and junior doctors 2010Derm handbook for medical students and junior doctors 2010
Derm handbook for medical students and junior doctors 2010Derma202
 

Más de Derma202 (7)

Histopathplogical photos
Histopathplogical photosHistopathplogical photos
Histopathplogical photos
 
Slide study from ETAS
Slide  study from ETASSlide  study from ETAS
Slide study from ETAS
 
Dermatology
DermatologyDermatology
Dermatology
 
ETAS_MCQ_14 plants and creatures of dermatologic significance
ETAS_MCQ_14 plants and creatures of dermatologic significanceETAS_MCQ_14 plants and creatures of dermatologic significance
ETAS_MCQ_14 plants and creatures of dermatologic significance
 
ETAS_MCQ_13 photobiology and photosensitivity disorders
ETAS_MCQ_13 photobiology and photosensitivity disordersETAS_MCQ_13 photobiology and photosensitivity disorders
ETAS_MCQ_13 photobiology and photosensitivity disorders
 
ETAS_MCQ_03 b genodermatoses
ETAS_MCQ_03 b genodermatosesETAS_MCQ_03 b genodermatoses
ETAS_MCQ_03 b genodermatoses
 
Derm handbook for medical students and junior doctors 2010
Derm handbook for medical students and junior doctors 2010Derm handbook for medical students and junior doctors 2010
Derm handbook for medical students and junior doctors 2010
 

Último

Introduction to Sports Injuries by- Dr. Anjali Rai
Introduction to Sports Injuries by- Dr. Anjali RaiIntroduction to Sports Injuries by- Dr. Anjali Rai
Introduction to Sports Injuries by- Dr. Anjali RaiGoogle
 
Glomerular Filtration rate and its determinants.pptx
Glomerular Filtration rate and its determinants.pptxGlomerular Filtration rate and its determinants.pptx
Glomerular Filtration rate and its determinants.pptxDr.Nusrat Tariq
 
Wessex Health Partners Wessex Integrated Care, Population Health, Research & ...
Wessex Health Partners Wessex Integrated Care, Population Health, Research & ...Wessex Health Partners Wessex Integrated Care, Population Health, Research & ...
Wessex Health Partners Wessex Integrated Care, Population Health, Research & ...Wessex Health Partners
 
POST NATAL EXERCISES AND ITS IMPACT.pptx
POST NATAL EXERCISES AND ITS IMPACT.pptxPOST NATAL EXERCISES AND ITS IMPACT.pptx
POST NATAL EXERCISES AND ITS IMPACT.pptxvirengeeta
 
PULMONARY EDEMA AND ITS MANAGEMENT.pdf
PULMONARY EDEMA AND  ITS  MANAGEMENT.pdfPULMONARY EDEMA AND  ITS  MANAGEMENT.pdf
PULMONARY EDEMA AND ITS MANAGEMENT.pdfDolisha Warbi
 
Let's Talk About It: To Disclose or Not to Disclose?
Let's Talk About It: To Disclose or Not to Disclose?Let's Talk About It: To Disclose or Not to Disclose?
Let's Talk About It: To Disclose or Not to Disclose?bkling
 
PNEUMOTHORAX AND ITS MANAGEMENTS.pdf
PNEUMOTHORAX   AND  ITS  MANAGEMENTS.pdfPNEUMOTHORAX   AND  ITS  MANAGEMENTS.pdf
PNEUMOTHORAX AND ITS MANAGEMENTS.pdfDolisha Warbi
 
call girls in Dwarka Sector 21 Metro DELHI 🔝 >༒9540349809 🔝 genuine Escort Se...
call girls in Dwarka Sector 21 Metro DELHI 🔝 >༒9540349809 🔝 genuine Escort Se...call girls in Dwarka Sector 21 Metro DELHI 🔝 >༒9540349809 🔝 genuine Escort Se...
call girls in Dwarka Sector 21 Metro DELHI 🔝 >༒9540349809 🔝 genuine Escort Se...saminamagar
 
Radiation Dosimetry Parameters and Isodose Curves.pptx
Radiation Dosimetry Parameters and Isodose Curves.pptxRadiation Dosimetry Parameters and Isodose Curves.pptx
Radiation Dosimetry Parameters and Isodose Curves.pptxDr. Dheeraj Kumar
 
METHODS OF ACQUIRING KNOWLEDGE IN NURSING.pptx by navdeep kaur
METHODS OF ACQUIRING KNOWLEDGE IN NURSING.pptx by navdeep kaurMETHODS OF ACQUIRING KNOWLEDGE IN NURSING.pptx by navdeep kaur
METHODS OF ACQUIRING KNOWLEDGE IN NURSING.pptx by navdeep kaurNavdeep Kaur
 
Presentation on General Anesthetics pdf.
Presentation on General Anesthetics pdf.Presentation on General Anesthetics pdf.
Presentation on General Anesthetics pdf.Prerana Jadhav
 
Informed Consent Empowering Healthcare Decision-Making.pptx
Informed Consent Empowering Healthcare Decision-Making.pptxInformed Consent Empowering Healthcare Decision-Making.pptx
Informed Consent Empowering Healthcare Decision-Making.pptxSasikiranMarri
 
PERFECT BUT PAINFUL TKR -ROLE OF SYNOVECTOMY.pptx
PERFECT BUT PAINFUL TKR -ROLE OF SYNOVECTOMY.pptxPERFECT BUT PAINFUL TKR -ROLE OF SYNOVECTOMY.pptx
PERFECT BUT PAINFUL TKR -ROLE OF SYNOVECTOMY.pptxdrashraf369
 
See the 2,456 pharmacies on the National E-Pharmacy Platform
See the 2,456 pharmacies on the National E-Pharmacy PlatformSee the 2,456 pharmacies on the National E-Pharmacy Platform
See the 2,456 pharmacies on the National E-Pharmacy PlatformKweku Zurek
 
Basic principles involved in the traditional systems of medicine PDF.pdf
Basic principles involved in the traditional systems of medicine PDF.pdfBasic principles involved in the traditional systems of medicine PDF.pdf
Basic principles involved in the traditional systems of medicine PDF.pdfDivya Kanojiya
 
Primary headache and facial pain. (2024)
Primary headache and facial pain. (2024)Primary headache and facial pain. (2024)
Primary headache and facial pain. (2024)Mohamed Rizk Khodair
 
History and Development of Pharmacovigilence.pdf
History and Development of Pharmacovigilence.pdfHistory and Development of Pharmacovigilence.pdf
History and Development of Pharmacovigilence.pdfSasikiranMarri
 
call girls in paharganj DELHI 🔝 >༒9540349809 🔝 genuine Escort Service 🔝✔️✔️
call girls in paharganj DELHI 🔝 >༒9540349809 🔝 genuine Escort Service 🔝✔️✔️call girls in paharganj DELHI 🔝 >༒9540349809 🔝 genuine Escort Service 🔝✔️✔️
call girls in paharganj DELHI 🔝 >༒9540349809 🔝 genuine Escort Service 🔝✔️✔️saminamagar
 
Pharmaceutical Marketting: Unit-5, Pricing
Pharmaceutical Marketting: Unit-5, PricingPharmaceutical Marketting: Unit-5, Pricing
Pharmaceutical Marketting: Unit-5, PricingArunagarwal328757
 
call girls in green park DELHI 🔝 >༒9540349809 🔝 genuine Escort Service 🔝✔️✔️
call girls in green park  DELHI 🔝 >༒9540349809 🔝 genuine Escort Service 🔝✔️✔️call girls in green park  DELHI 🔝 >༒9540349809 🔝 genuine Escort Service 🔝✔️✔️
call girls in green park DELHI 🔝 >༒9540349809 🔝 genuine Escort Service 🔝✔️✔️saminamagar
 

Último (20)

Introduction to Sports Injuries by- Dr. Anjali Rai
Introduction to Sports Injuries by- Dr. Anjali RaiIntroduction to Sports Injuries by- Dr. Anjali Rai
Introduction to Sports Injuries by- Dr. Anjali Rai
 
Glomerular Filtration rate and its determinants.pptx
Glomerular Filtration rate and its determinants.pptxGlomerular Filtration rate and its determinants.pptx
Glomerular Filtration rate and its determinants.pptx
 
Wessex Health Partners Wessex Integrated Care, Population Health, Research & ...
Wessex Health Partners Wessex Integrated Care, Population Health, Research & ...Wessex Health Partners Wessex Integrated Care, Population Health, Research & ...
Wessex Health Partners Wessex Integrated Care, Population Health, Research & ...
 
POST NATAL EXERCISES AND ITS IMPACT.pptx
POST NATAL EXERCISES AND ITS IMPACT.pptxPOST NATAL EXERCISES AND ITS IMPACT.pptx
POST NATAL EXERCISES AND ITS IMPACT.pptx
 
PULMONARY EDEMA AND ITS MANAGEMENT.pdf
PULMONARY EDEMA AND  ITS  MANAGEMENT.pdfPULMONARY EDEMA AND  ITS  MANAGEMENT.pdf
PULMONARY EDEMA AND ITS MANAGEMENT.pdf
 
Let's Talk About It: To Disclose or Not to Disclose?
Let's Talk About It: To Disclose or Not to Disclose?Let's Talk About It: To Disclose or Not to Disclose?
Let's Talk About It: To Disclose or Not to Disclose?
 
PNEUMOTHORAX AND ITS MANAGEMENTS.pdf
PNEUMOTHORAX   AND  ITS  MANAGEMENTS.pdfPNEUMOTHORAX   AND  ITS  MANAGEMENTS.pdf
PNEUMOTHORAX AND ITS MANAGEMENTS.pdf
 
call girls in Dwarka Sector 21 Metro DELHI 🔝 >༒9540349809 🔝 genuine Escort Se...
call girls in Dwarka Sector 21 Metro DELHI 🔝 >༒9540349809 🔝 genuine Escort Se...call girls in Dwarka Sector 21 Metro DELHI 🔝 >༒9540349809 🔝 genuine Escort Se...
call girls in Dwarka Sector 21 Metro DELHI 🔝 >༒9540349809 🔝 genuine Escort Se...
 
Radiation Dosimetry Parameters and Isodose Curves.pptx
Radiation Dosimetry Parameters and Isodose Curves.pptxRadiation Dosimetry Parameters and Isodose Curves.pptx
Radiation Dosimetry Parameters and Isodose Curves.pptx
 
METHODS OF ACQUIRING KNOWLEDGE IN NURSING.pptx by navdeep kaur
METHODS OF ACQUIRING KNOWLEDGE IN NURSING.pptx by navdeep kaurMETHODS OF ACQUIRING KNOWLEDGE IN NURSING.pptx by navdeep kaur
METHODS OF ACQUIRING KNOWLEDGE IN NURSING.pptx by navdeep kaur
 
Presentation on General Anesthetics pdf.
Presentation on General Anesthetics pdf.Presentation on General Anesthetics pdf.
Presentation on General Anesthetics pdf.
 
Informed Consent Empowering Healthcare Decision-Making.pptx
Informed Consent Empowering Healthcare Decision-Making.pptxInformed Consent Empowering Healthcare Decision-Making.pptx
Informed Consent Empowering Healthcare Decision-Making.pptx
 
PERFECT BUT PAINFUL TKR -ROLE OF SYNOVECTOMY.pptx
PERFECT BUT PAINFUL TKR -ROLE OF SYNOVECTOMY.pptxPERFECT BUT PAINFUL TKR -ROLE OF SYNOVECTOMY.pptx
PERFECT BUT PAINFUL TKR -ROLE OF SYNOVECTOMY.pptx
 
See the 2,456 pharmacies on the National E-Pharmacy Platform
See the 2,456 pharmacies on the National E-Pharmacy PlatformSee the 2,456 pharmacies on the National E-Pharmacy Platform
See the 2,456 pharmacies on the National E-Pharmacy Platform
 
Basic principles involved in the traditional systems of medicine PDF.pdf
Basic principles involved in the traditional systems of medicine PDF.pdfBasic principles involved in the traditional systems of medicine PDF.pdf
Basic principles involved in the traditional systems of medicine PDF.pdf
 
Primary headache and facial pain. (2024)
Primary headache and facial pain. (2024)Primary headache and facial pain. (2024)
Primary headache and facial pain. (2024)
 
History and Development of Pharmacovigilence.pdf
History and Development of Pharmacovigilence.pdfHistory and Development of Pharmacovigilence.pdf
History and Development of Pharmacovigilence.pdf
 
call girls in paharganj DELHI 🔝 >༒9540349809 🔝 genuine Escort Service 🔝✔️✔️
call girls in paharganj DELHI 🔝 >༒9540349809 🔝 genuine Escort Service 🔝✔️✔️call girls in paharganj DELHI 🔝 >༒9540349809 🔝 genuine Escort Service 🔝✔️✔️
call girls in paharganj DELHI 🔝 >༒9540349809 🔝 genuine Escort Service 🔝✔️✔️
 
Pharmaceutical Marketting: Unit-5, Pricing
Pharmaceutical Marketting: Unit-5, PricingPharmaceutical Marketting: Unit-5, Pricing
Pharmaceutical Marketting: Unit-5, Pricing
 
call girls in green park DELHI 🔝 >༒9540349809 🔝 genuine Escort Service 🔝✔️✔️
call girls in green park  DELHI 🔝 >༒9540349809 🔝 genuine Escort Service 🔝✔️✔️call girls in green park  DELHI 🔝 >༒9540349809 🔝 genuine Escort Service 🔝✔️✔️
call girls in green park DELHI 🔝 >༒9540349809 🔝 genuine Escort Service 🔝✔️✔️
 

ETAS_MCQ_09 pediatric dermatology

  • 1. Pediatric Dermatology 1) A full term newborn develops erythematous, indurated plaques on the upper back. Which of the following tests should be performed? A. Thyroid function test B. CalciumCorrect Choice C. Blood culture D. Complete blood count E. Alkaline phosphatase The most likely diagnosis is subcutaneous fat necrosis of the newborn. Hypercalcemia may result and serum calcium levels should be monitored up to 4 weeks after resolution of the skin 2) What is the most common tumor associated with this condition? A. Trichoadenoma B. Trichoblastoma Correct Choice C. Syringocystadenoma papilliferum D. Basal cell carcinoma E. Sebaceous carcinoma Recent reviews have identified trichoblastoma as the most common tumor arising within nevus sebaceous 3) Which finding is associated with Pachydermoperiostosis: A. Mucosal keratoses B. Cutis marmorata C. Cutis verticis gyrataCorrect Choice D. Palmoplantar hyperkeratosis E. Osteopathia striata The scalp change seen in pachydermoperiostosis is cutis verticis gyrata 4) An infant presents with the lesion depicted in the photo. What information do you provide the parents regarding this condition? A. The lesion will persist and may grow further Correct Choice B. The lesion is associated with a XO karyotype C. The lesion will respond to laser therapy D. The lesion will go through rapid growth followed by stabilization and regression E. The lesion will resolve with antifungal therapy The lesion represented in the photo is an arteriovenous vascular malformation. These lesions do not typically regress 5) Which of the following is most likely associated? 1
  • 2. A. No additional abnormalityCorrect Choice B. Deafness C. Bifid rib D. Alopecia E. Coarctation of the aorta The majority of patients with accessory tragii do not have any associated abnormalities. Multiple anomalies, including deafness and midline defects, have been reported but are rare Trichothiodystrophy classically includes which of the following findings: A. Trichorrhexis nodosa B. Pili torti C. Trichostasis D. TrichoschisisCorrect Choice E. Trichorrhexis invaginata Trichothiodystrophy or PIBIDS is associated with the “tiger-tail” abnormality of trichoschisis 7) A 3 month-old girl with multiple hemangiomas along her right jaw is at increased risk for: A. Subglottic hemangiomaCorrect Choice B. Hearing defecits C. Underlying bone abnormalities D. Bleeding complications E. Oral obstruction Hemangiomas are benign vascular tumors which have proliferating phase and then a spontaneous involution phase. Depending upon the location, some hemangiomas may have more long term sequelae such as scarring or structural malformation, and rarely, consumptive coagulopathy. Infants with hemangiomas of the “beard” distribution should be evaluated for subglottic hemangiomas which may lead to airway obstruction A 3 month-old girl with multiple hemangiomas along her right jaw is at increased risk for: A. Subglottic hemangiomaCorrect Choice B. Hearing defecits C. Underlying bone abnormalities D. Bleeding complications E. Oral obstruction Hemangiomas are benign vascular tumors which have proliferating phase and then a spontaneous involution phase. Depending upon the location, some hemangiomas may have more long term sequelae such as scarring or structural malformation, and rarely, consumptive coagulopathy. Infants with hemangiomas of the “beard” distribution should be evaluated for subglottic hemangiomas which may lead to airway obstruction 8) In a child with zinc deficiency, yet normal or near normal zinc levels, which test could be a valuable adjunctive test? A. Niacin 2
  • 3. B. Iron C. Alkaline phosphataseCorrect Choice D. Magnesium E. Manganese The diagnosis of zinc deficiency should be consideredi in at-risk individuals with acral or periorificial dermatitis. Chronic diaper dermatitis in an infant should lead to the evaluation for zinc deficiency. A low serum zinc level can usually confirm the diagnosis. If normal or near normal a low serum alkaline phosphatase, a zinc-dependent enzyme, may be a valuable adjunctive test 9) Posterior auricular adenopathy is a common feature of which exanthem: A. Rubella Correct Choice B. Scarlet fever C. Measles D. Erythema infectiosum E. Mumps Lymphadenopathy with 1-7 days of malaise is commonly described with rubella infection. The rose- pink macular rash follows the prodrome 10) What is best the test to confirm a diagnosis? A. Anti Ro antibody B. Biopsy C. Gram stain D. KOHCorrect Choice E. NA antibody The picture depicts tinea faceii. A KOH exam should be performed to look for hyphae 11) What is the diagnosis? A. Neonatal cphalic pustulosisCorrect Choice B. Transient neonatal pustular melanosis C. Erythema toxicum neonatorum D. Acropustulosis of infancy E. Miliaria In neonatal cephalic pustulosis (neonatal acne), papules and pustules are present but comedones are absent. Spontaneous remission occurs 12) Which of the following is a potential complication of subcutaneous fat necrosis of the newborn? A. Cholestrol clefts in fat cells B. Acute renal failure C. Hepatitis 3
  • 4. D. HypercalcemiaCorrect Choice E. Elevated uric acid levels Subcutaneous fat necrosis of the newborn is a rare condition characterized by firm, reddish or purple nodules which appear on the arms, thighs, buttocks, back and cheeks. This condition is believed to result from cold injury. Crystallization occurs within the lipocytes, and this can be seen on histopathology with an associated granulomatous reaction. This is thought to occur due to the higher melting point of neonatal fat, which contains more saturated fatty acids. Onset of this condition occurs within the first two weeks of life and usually resolved over a period of weeks to months. Occasionally, lesions heal with atrophy. Hypercalcemia is an infrequent complication of subcutaneous fat necrosis of the newborn. Associated symptoms can include irritability, weight loss, vomiting and failure to thrive. Repeated serum calcium tests are advised until one month after all the cutaneous lesions have resolved 13) What is the most appropriate management of this condition? A. Observation Correct Choice B. Pulsed dye laser C. Embolization D. Oral corticosteroids E. MRI This V-shaped vascular lesion is nevus flammeus neonatorum. Most lesions will fade over time without intervention 14) An infant with failure to thrive has multiple xanthomas on skin exam and foamy histiocytes on bone marrow biopsy. Your diagnosis is: A. Niemann-Pick diseaseCorrect Choice B. Fabry’s disease C. Tay-Sach’s disease D. Hunter’s syndrome E. Gaucher’s disease Niemann-Pick disease is an autosomal recessive disease caused by mutations in sphingomyelin phosphodiesterase-1. Patients with Type A Niemann-Pick disease may have xanthomas, progressive psychomotor deterioration, hepatosplenomegaly, blindness, cherry red spots, and deafness 15) An infant presents with yellowish-brown, crusted papules with petechiae in a seborrheic distribution. Which of the following statements about this entity is likely to be true? A. Maternal-fetal transmission most likely occured in the peripartum period B. This presentation occurs when the infant is weaned off of breast milk C. CD1+, S100+ cells with comma-shaped nuclei should be seen on biopsyCorrect Choice D. An autosomal recessive defect in holocarboxylase synthetase is the cause E. This presentation is consistent with Jacquet's dermatitis This infant has Langerhans Cell Histiocytosis (Letterer-Siwe disease). Multisystem involvement may be present. Jacquet's Erosive Dermatitis presents with severe erosive papules in the diaper region, and is multifactorial in etiology (yeast, irritant dermatitis, moisture). Acrodermatitis enteropathica (zinc deficiency) presents with brown, orange crusted plaques with vesicles and bullae, especially in 4
  • 5. perineal and perioral areas and distal extremities. The inherited form occurs when the infant is weaned off of breast milk. Biotin deficiency presents similarly; the neonatal form is attributed to an AR defect in holocarboxylase synthetase 16) What is the best therapeutic option? A. Topical mupirocin B. Oral prednisone C. Topical immune modulator D. Oral cephalexin E. Oral acyclovirCorrect Choice The condition shown in an example of eczema herpeticum, also called Kaposi varicelliform eruption. It occurs when pre-existing dermatitis becomes superinfected with a viral infection, most commonly atopic herpes infection of atopic dermatitis. The initial treatment of choice is antiviral medications 17) Multiple cylindromas are associated with: A. Trichoepitheliomas Correct Choice B. Cowden syndrome C. Pilomatrichomas D. Carney complex E. Myotonic dystrophy The Brooke-Spigler syndrome is defined by the presence of multiple trichoepitheliomas and cylindromas 18) This condition may be associated with which of the following: A. Perinatal trauma B. Hyperlipidemia C. Hypercalcemia D. Epidermolysis bullosa dystrophica Correct Choice E. Vitiligo Aplasia cutis congenita may be associated with dystrophic epidermolysis bullosa (Bart syndrome 19) Neutrophils are characteristically seen on smears from pustules of which of the following transient benign neonatal rashes? A. Acne neonatorum B. Erythema toxicum neonatorum C. Acne neonatorum and transient neonatal pustular melanosisCorrect Choice D. Infantile acne E. Transient neonatal pustular melanosis Neutrophils are seen on smears from pustules of acne neonatorum and transient neonatal pustular melanosis. Eosinophils are characteristically seen on smears from erythema toxicum neonatorum 5
  • 6. 20) Which of the following is characteristic of Wooly Hair Nevus: A. Ocular abnormalities may be associated B. Sponteneous improvement never occurs C. Typically occurs in the 5th decade D. There are no nevi associated with this hair abnormality E. It is hereditaryCorrect Choice Wooly Hair Nevus is the presence of Negroid hair on the scalp of persons of non-Negroid inheritance. The unruly hair presents at birth or in infancy, usually as a solitary problem inherited in an autosomal dominant fashion 21) What is the best treatment option? A. Topical tretinoinCorrect Choice B. Oral acyclovir C. Topical ketoconazole D. Observation E. Oral cephalexin The lesions shown in the photo are comedones of infantile acne. Topical tretinoin would be the best treatment option 22) Which of the following can present as collodion baby? A. X-linked ichthyosis B. Bullous congenital ichthyosiform erythroderma C. Sjogren-Larsson syndrome D. Lamellar ichthyosisCorrect Choice E. Ichthyosis vulgaris The most common presentation of collodion baby is lamellar ichthyosis, followed by congenital ichthyosiform erythroderma. Patients with ichthyosis vulgaris and x-linked ichthyosis are normal at birth. Bullous congenital ichthyosiform erythroderma or epidermolytic hyperkeratosis presents with widespread bullae, erthroderma, and denuded skin. Sjogren-Larsson presents with generalized ichthyosis and erythroderma in infancy. It is important to know the at-birth presentations of all the disorders of cornification 23) A 2 day-old full term neonate develops blotchy erythematous macules with small central pustules over the upper trunk and extremities. A gram stain reveals predominantly eosinophils. What is the most likely diagnosis? A. Incontinentia pigmenti B. Erythema toxicum neonatorum Correct Choice C. Urticaria pigmentosa D. Transient neonatal pustular melanosis 6
  • 7. E. Miliaria Erythema toxicum neonatorum is a very common eruption in healthy newborns. A gram stain reveals sterile pustules containing eosinophils 24) An eight year-old boy presents with pink, flushed cheeks and a low-grade fever. The following eruption then appeared. What is the most likely etiology? A. Paramyxovirus B. Togavirus C. Parvovirus Correct Choice D. HHV6 E. Group A streptococcus This condition, also called Fifths Disease or erythema infectiosum, is caused by parvovirus B19, a single-stranded DNA virus. There are a few clinical presentations including, diffuse lacy rash on the trunk that spreads gradually toward the distal extremities, papular-pruritic "gloves-and-socks" syndrome and aplastic crisis which does not have concomitant rash 25) What is the best test to confirm a diagnosis? A. Gram stain B. KOH Correct Choice C. Biopsy D. Anti Ro antibody E. DsDNA antibody The picture depicts tinea faceii. A KOH exam should be performed to look for hyphae 26) Which presentation of psoriasis is more common in children: A. Acrodermatitis continua of Hallopeau B. Guttate psoriasisCorrect Choice C. Keratoderma blennorragica D. Pustular psoriasis E. Erythrodermic psoriasis The majority of cases of guttate psoriasis occur in persons under the age of 30 27) This patient had significantly elevated serum CPK. The likely diagnosis is: A. Dermatomyositis Correct Choice B. Lupus erythematosus C. Atopic dermatitis D. Psoriasis E. Lichen planus 7
  • 8. The answer is dermatomyositis with the heliotrope color and distribution of erythema or violaceous color. The skin over the metacarpal and proximal interphalangeal joints can become inflamed and erythematous forming Gottron’s papules ٢٨) Menkes kinky hair syndrome is associated most commonly with: A. Trichorrhexsis invaginata B. Pili torti Correct Choice C. Pili multigemini D. Plica neuropathica E. Trichostasis spinulosa The X-linked recessive Menkes kinky hair syndrome is associated with multiple hair shaft abnormalities, most characteristically, pili torti 29) What disorder is associated with a defect in LMX1B? A. Lhermitte-Duclos syndrome B. Chediak-Higashi syndrome C. Steatocystoma D. Nail patella syndrome Correct Choice E. Monilithrix A LMX1G gene defect is associated with nail patella syndrome 30) Schimmelpenning-Feuerstein-Mims syndrome may be associated with which of the following: A. Hypophosphatemic ricketsCorrect Choice B. Polyostotic fibrous dysplasia C. Osteopathia striata D. Chondrodysplasia punctata E. Osteopokilosis Epidermal nevus syndrome, also known as Schimmelpenning-Feuerstein-Mims syndrome, may be associated with hypophospatemic, vitamin D resistant rickets 31) Variants of xeroderma pigmentosum are due to all of the following defects except: A. Endonuclease B. Helicase C. Thymidine kinase Correct Choice D. Postrepliction repair E. Nucleotide excision repair The photosensitivity of xeroderma pigmentosum is caused by defect in DNA repair mechanisms. Thymidine kinase is not affected 32) “Slapped cheeks” followed by a lacy eruption on extremities: 8
  • 9. A. ssRNA virus B. dsDNA virus C. ssDNA virus Correct Choice D. Streptococcus E. dsRNA virus The erythematous eruption on the cheeks precedes the generalized lacy rash of Fifth disease. The causative agent is parvovirus B19, a single stranded DNA virus 33) Ankyloblepharon filiforme adnatum is seen with a defect in: A. p63 Correct Choice B. Plakophilin C. Desmoglein D. C-kit E. Plakoglobin The ectodermal-clefting syndromes are caused by a defect in the p63 gene. Specifically, AEC or Hay-Wells syndrome is comprised of ankyloblepharon filiforme adnatum, ectodermal dysplasia and clefting 34) Pachyonychia congenita type 2 is most commonly associated with which of the following: A. Deafness B. Aplastic nails C. Increased risk of malignancy D. Natal teethCorrect Choice E. Poikiloderma Pachyonychia congenita type 2 may be associated with natal teeth and steatocystoma. Pachyonychia congenita type 1 is associated with benign leukoplakia 35) The risk of fetal death with intrauterine parvovirus infection may occur with infection in which trimester: A. First B. Third C. Second D. First, Second and Third Correct Choice E. None of these answers are correct Fetal hydrops may occur with parvovirus infection during all three trimesters although the greatest risk is during the second trimester. Congenital anomalies are not a feature 36) Individuals with which of the following syndromes characteristically present with photosensitivity, mental retardation, a "wizened" appearance, "bird-headed" facies, and "Mickey Mouse" ears? 9
  • 10. A. Rothmund-Thomson Syndrome B. Werner Syndrome C. Tay Syndrome D. Cockayne SyndromeCorrect Choice E. Hutchinson-Gilford Progeria Syndrome Cockayne Syndrome is AR, caused by defective excision repair, cross-complementing group 8 gene (ERCC8). It presents with photosensitivity, mental retardation, and cachectic dwarfism. Patients have a characteristic "wizened" appearance, "bird-headed" facies, and "Mickey Mouse" ears. Cataracts, deafness, pigmentary retinopathy, dental caries, and skeletal, GU, and endocrine abnormalities may be seen 37) What is the most common tumor associated with this condition? A. Sebaceous carcinoma B. Trichoadenoma C. Trichoblastoma Correct Choice D. Basal cell carcinoma E. Syringocystadenoma papilliferum Recent reviews have identified trichoblastoma as the most common tumor arising within nevus sebaceous 38) What the most likely diagnosis? A. Wiskott-Aldrich syndrome B. Atopic dermatitis Correct Choice C. Rud syndrome D. Lamellar ichthyosis E. Keratosis pilaris Ichythosis vulgaris and hyperlinear palms are both independent minor criteria for the diagnosis of atopic dermatitis in both children and infants 39) A boy is noted at birth to have coarse scales over his trunk and extremities. The face, palms, soles and flexures are spared. What is the least likely association? A. Ectropion Correct Choice B. Corneal opacities C. Cryptorchidism D. Neurologic abnormality E. Prolonged maternal labor X-linked ichthyosis is characterized by small, dark, firmly adherent scales accentuated on the sides of the neck and trunk. The face, palms, soles, antecubital and popliteal flexures are generally spared. Associated extracutaneous findings include corneal opacities (50%), undescended testes (20%), and prolonged maternal labor (usual). Neurological or mental retardation are rare but documented associations as XLI can be associated with a contiguous gene syndrome with Kallman syndrome, mental retardation and X linked recessive chrondrodysplasia punctata 10
  • 11. 40) Which of the following is not a major criterion for Kawasaki’s disease: A. Palmoplantar erythema > desquamation B. Cardiac aneurysm Correct Choice C. Fever >5 days D. Strawberry tongue/ red lips E. Cervical adenopathy Cardiac aneurysm is a serious complication of Kawasaki’s disease. However, as the cardiovascular manifestations generally present 1 –5 months after presentation, they are not criteria for diagnosis 41) Koplik spots typically appear: A. At the same time as the exanthem B. 1 week after the exanthem C. 8 weeks after the exanthem D. 6 weeks after the exanthem E. Before the exanthem Correct Choice The enanthem of measles precedes the morbilliform eruption. 42) Pastia’s lines are characteristic for which eruption: A. Kawasaki’s disease B. Scarlet fever Correct Choice C. Rubella D. Mumps E. Measles Pastia’s lines are defined as a linear accentuation of the erythematous sandpaper rash within the flexures. This is classically described with scarlet fever 43) Which enanthem is most commonly seen in association with Exanthem subitum? A. Koplik spots B. Red strawberry tongue C. Red macules and streaks on the soft palateCorrect Choice D. Palatal erosions E. Chapped lips; dry, red mucosa Exanthem subitum (roseola or Sixth disease) is caused by HHV6, a dsDNA virus. It presents with high fever for several days followed by an exanthem of erythematous macules and papules on the trunk that begins as the fever ends. An associated enanthem of red macules/streaks on the soft palate may be seen. Koplik spots are seen in measles; the red strawberry tongue (following the white strawberry tongue) is seen in Scarlet fever; chapped lips and dry, red mucosa may be seen in Kawasaki's disease; palatal erosions may be seen in Papular-purpuric gloves and socks syndrome 11
  • 12. 44) An 8 year-old boy presents with pink, flushed cheeks and a low-grade fever. On week later, the following lacy eruption appeared. What is the most likely etiology? A. Togavirus B. HHV6 C. Group A streptococcus D. Paramyxovirus E. Parvovirus Correct Choice Fifth disease is caused by Parvovirus B19. Most cases start with prodrome of fever, malaise, headache and rhinorrhea. Cutaneous reaction follows approximately 5-7 days later with erythema of the cheeks ("slapped cheeks") and reticulate rash of the trunk and extremities 45) An infant presents with red-purple, granulomatous nodules occurring in the diaper area. The etiology is secondary to local irritation, maceration and Candida albicans. What is the most likely diagnosis? A. Langerhans cell histiocytosisc. B. Biotin deficiency C. Seborrheic dermatitis D. Psoriasis E. Granuloma gluteale infantum Correct Choice The etiology of granuloma gluteale infantum is multifactorial, resulting from the unique environment of the diaper area. Treatment consists of topical antifungal agents, barrier creams, and anti- inflammatory agents as needed 46) The Carney complex is associated with a defect in: A. LYST B. PTEN C. PRKAR1A Correct Choice D. MASH2 E. MLH1 A PRKAR1A gene defect is associated with the Carney complex 47) What syndrome is the disorder shown in the photo associated with? A. Noonan syndrome Correct Choice B. Bloom syndrome C. Turner syndrome D. Down syndrome E. Griscelli syndrome The photo shows keratosis pilaris atrophicans faceii and surgically corrected ptosis. Both of these findings are associated with Noonan syndrome 12
  • 13. 48) A 6 month-old presents with orange-brown crusted plaques around the mouth and groin. Several bullae are present on the fingers and toes. Which of the following laboratory values is likely to be abnormal? A. Calcium B. Hematocrit C. Alkaline phosphatase Correct Choice D. Platelet count E. ALT The most likely diagnosis is acrodermatitis enteropathica. Alkaline phosphatase is a zinc dependant enzyme that is decreased in response to low serum zinc levels 49) Which of the following should be the next step in the management of this patient? A. spine x-ray B. Administration of oral antibiotics C. Cardiac evaluationCorrect Choice D. Barium swallow E. Pulmonary function studies This patient must be evaluated thoroughly for PHACES syndrome. PHACES is an acronym for Posterior fossa malformations (Dandy-Walker malformation is most common), Hemangiomas, Arterial anomalies, Coarctation of the aorta, Eye abnormalities, and Sternal cleft defects. This patient should have a complete cardiac evaluation, neuroimaging, and ophthalmologic exam. If the facial hemangioma involves the beard area, this may indicate laryngeal involvement and appropriate imaging and evaluation is mandated. Systemic steroids at high doses (5 mg/kg) are usually administered. If caught early, the sequelae of PHACES syndrome can be minimized 50) The association of Port-wine stains on a limb with soft tissue swelling with or without bony over growth is: A. Sturge-Weber syndrome B. Goldenhar's syndrome C. Klippel-Trenaunay syndromeCorrect Choice D. Bannayan-Riley-Ruvalcaba syndrome E. Proteous syndrome The association of port-wine stain on a limb with soft tissue swelling with or without bony overgrowth is Klippel-Trenuanay syndrome. Klippel-Trenaunay syndrome is characterized by the Triad of port-wine malformations in association with deep venous system malformations, superficial varicosities, and bony and soft tissue hypertrophy. Sturge-Weber syndrome has 2 essential components: Facial port-wine stain and homolateral leptomeningeal angiiomattosis. The port wine stain most commonly involves the areas innervated by the ophthalmic(V1) and maxillary (V2) divisions of the trigeminal nerve. Complications of leptomeningeal angiomatosis are epilepsy, mental retardation, and occasionally, contralateral hemiplegia. Proteus Syndrome is characterized by vascular malformations including nevus flammeus, hemihypertrophy, macrodactyly, verrucous epidermal nevus, soft-tissue subcutaneous masses, and cerebriform overgrowth of the plantar surface. Babbyan-Riley-Ruvalcaba syndrome may include multiple cutaneous and visceral venous, capillary, and lympathtic malformations, macroephaly, pseudopapilledema, systemic lipoangiomatosis, spotted pigmentation of the penis, hamartomatous intestinal polyps, and rarely trichilemmonmas. (multiple subcutaneous lipomas as well as acanthosis nigricans). 13
  • 14. 51) The most common age group for papular-purpuric gloves and socks syndrome is: A. Newborns B. 6-10 year olds C. Toddlers D. Adolescents Correct Choice E. Elderly This unique presentation of parvovirus infection typically occurs in adolescents and young adults 52) What is the most likely diagnosis? A. Traction alopecia B. Nevus sebaceous C. Alopecia areata D. Tinea capitis E. Aplasia cutis congenita Correct Choice Aplasia cutis congenita is characterized by the absence of a portion of skin, most commonly presenting as a solitary defect on the scalp, but sometimes it may occur as multiple lesions. The lesions are non-inflammatory and well demarcated appear as an atrophic, membranous, ulcerated area with alopecia. The condition may be associated with other physical anomlies 53) POEMS syndrome is associated with which of the following: A. M proteinCorrect Choice B. Odontogenic cysts C. Saddle nose deformity D. Premature aging E. Eye abnormalities The acronym POEMS stands for polyneuropathy, organomegaly, endocrinopathy, M-protein, and skin changes 54) Osteopathia striata is found in which disorder? A. Gorlin's syndrome B. Neurofibromatosis I C. McCune-Albright syndrome D. Buschke-Ollendorff syndrome E. Focal dermal hypoplasiaCorrect Choice Osteopathia striata (vertical striations in the metaphysis of long bones on x-ray) is seen is greater than 80% of cases of focal dermal hypoplasia (or Goltz syndrome). Polyostotic fibrous dysplasia with recurrent fractures is seen in McCune-Albright syndrome. Sphenoid wing dysplasia and thinning of long bone cortex is found in neurofibromatosis I. Osteopoikilosis is an asymptomatic x-ray finding in patients with Buschke-Ollendorf syndrome. Osteopoikilosis reflects ectopic calcification that does 14
  • 15. not increase risk of fracture. Bifid ribs, vertebral fusion/Sprengel deformity of the spine, and kyphoscoliosis can be seen in basal cell nevus syndrome (Gorlin's syndrome). 55) Which of the following is a ssDNA virus: A. Parvovirus Correct Choice B. Parapox C. Herpesvirus D. Picornovirus E. Adenovirus Parvovirus is the only ssDNA virus listed 56) Which treatment choice would be contraindicated in a one-year old child who presnents with monomorphous, nonpruritic flat-topped papules on the face, buttocks, extremities, palms and soles? A. Observation B. Aspirin C. CorticosteroidsCorrect Choice D. Acetaminophen E. Hydration Gianotti-Crosti or papular acrodermatitis of childhood is associated with a variety of viral infections. Patients have a typical cutaneous manifestation, low-grade fever, mild lymphadenopathy and diarrhea. Corticosteroids should be avoidedas they may have an adverse effect 57) The standard of care of patients with acute Kawasaki’s disease is: A. Supportive care B. Aspirin and IVIGCorrect Choice C. Penicillin D. Acetaminophen and IVIG E. Prednisone Kawasaki disease, also called mucocutaneous lymph node syndrome, is an acute febrile disorder based on the clinical criteria of changes in peripheral extremities, polymorphous exanthema, conjunctival injection without exudates, changes in the lips or oral cavity, acute cervical lymphadenopathy. Fever must be present, lasting more than 5 days. Treatment is aimed to prevent coronary aneurysms and myocardial infarction. Treatment for acute Kawasaki disease is intravenous immunoglobulin 2 g/kg over 10-12 hours and aspirin therapy 58) A newborn has a nodule over his lumbar spine. Skin biopsy reveals a lipoma. The next appropriate step is: A. Genetic testing B. Observation C. Imaging studyCorrect Choice D. Malignancy work up 15
  • 16. E. Excision of the lesion The skin can provide an important clue to the presence of an underlying neural tube defect, such as meningomyelocele and encephalocele. Cutaneous lesions along the midline of the spine should always prompt consideration of this possibility. Although, midline neural tube defects are uncommon, early recognition and diagnosis of a spinal dysraphism can have important implications for early surgical correction and minimizing loss of neurologic function. Clues to the diagnosis include a midline dimple, tuft of hair, lipoma, or vascular lesion. In these instances, imaging studies (MRI, CT, ultrasound) should be promptly initiated 59) An infant presents with the lesion depicted in the photo. Which of the following is least likely? A. Dandy-Walker malformation B. Congenital cataracts C. Sternal clefting D. Supraumbilical raphe E. Seizure disorder Correct Choice Extensive facial hemangiomas are a component of the PHACES syndrome. Seizure disorder has not been described 60) A 24 month-old infant presents with yellowish-brown, crusted papules with petechiae in a seborrheic distribution. A biopsy is done to confirm a diagnosis. Which histologic picture is most likely? A. CD1-, S100- cells with reniform nuclei B. Mixed cellular infiltrate in a “ball and claw” pattern C. Foamy histiocytes with Touton giant cells D. Superficial perivascular infiltrate with mild spongiosis and neutrophil containing scale crust E. CD1+, S100+ cells with reniform nuclei Correct Choice Langerhans cells are CD1 and S100 positive. The nuclei are described as kidney shaped, or reniform 61) What is the function of the gene which is defective in ataxia-telangiectasia? A. Pathway of cholesterol biosynthesis B. Cross-linking of structural proteins in the protein and lipid envelope of the upper epidermis C. Gap junction protein D. DNA repair proteinCorrect Choice E. Tumor supressor protein The defective gene is ataxia-telangiectasia (Louis-Bar syndrome) is the ATM gene, which is responsible for DNA repair, especially after ionizing radiation. Tumor suppressor genes mutations are responsible for basal cell nevus syndrome, xeroderma pigmentosum, Muir-Torre syndrome, dyskeratosis congenital, Gardner syndrome, Peutz-Jeghers syndrome, Cowden syndrome, and MEN syndromes. Connexins are gap junction proteins that are responsible for intercellular communication and signaling. Mutations in connexins are responsible for Vohwinkel syndrome and erythrokeratoderma variabilis. Mutations in the cholesterol biosynthesis pathways cause CHILD syndrome and Conradi-Hunermann syndrome. Tranglutaminase 1 (TGM 1) is involved in the normal cross-linking of structural proteins in the protein and lipid envelope of the upper epidermis. TGM 1 is mutated in lamellar ichthyosis and congenital ichthyosiform erythroderma 16
  • 17. 62) What is the diagnosis? A. Rubella B. Contact dermatitis C. Unilateral laterothoracic exanthem Correct Choice D. Urticaria E. Roseola The child in the photo depicts the “Statue of Liberty” sign of unilateral laterothoracic exanthem 63) The most appropriate treatment of a 2 month-old with a hemangioma involving the lateral canthus is: A. Observation and reassurance of the parents B. Pulsed dye laser C. Surgical excision D. Interferon-alpha E. Systemic corticosteroidsCorrect Choice Since hemangiomas characteristically follow a course of proliferation followed by spontaneous involution, many of the lesions can be followed with conservative management including observation. However, indications for treatment of hemangiomas include obstruction of vital function, high-output cardiac failure, ulceration, infection, diaper area location and location on the face. Aggressive treatment is indicated for a hemangioma located on the lateral canthus since two of the above indications apply. Importantly, obstruction of the visual field may impair development of the visual cortex in an infant. Oral glucocorticoids are the mainstay of treatment for hemangiomas, with 30 to 60% of lesions responsive to therapy. Alternative treatments for problematic hemangiomas include intralesional steroids, interferon alpha-2a, and pulsed dye laser 64) Neonatal acne is associated with species of which organism: A. Streptococcus B. Candida C. MalasseziaCorrect Choice D. Staphylococcus E. Propionibacterium Acne which develops within the first 30 days of life is termed neonatal acne. Neonatal acne has a predilection for the face, chest, back and groin appearing as small, discrete papules at 2 to 4 weeks of age, and persisting for up to 8 months. As these lesions are self-resolving, no treatment is necessary, though 2.5% benzoyl peroxide may hasten resolution. Neonatal acne is quite common and is postulated to occur as a result of hyperplasia of premature sebaceous glands coupled with transient increases in circulating androgens. More recent data suggests that Malassezia species may be implicated at etiologic factors in neonatal acne. These organisms have been cultured from the skin of affected patients, though their exact role in unclear 65) What is the most likely diagnosis? A. Cutaneous T-cell lymphoma B. Psoriasis 17
  • 18. C. Contact dermatitisCorrect Choice D. Lichen planus E. Pityriasis rosea This periumbilical eruption is classic for a contact nickel allergy. The metal snaps on pants are the cause in this case 66) What is the best therapeutic option? A. Oral cephalexin B. Oral prednisone C. Oral acyclovir Correct Choice D. Topical mupirocin E. Topical immune modulator The condition shown is eczema herpeticum. The most appropriate treatment is oral acyclovir 67) The most likely etiology of Jacquet’s diaper dermatitis is: A. Multifactorial Correct Choice B. Candida C. Trichophyton rubrum D. Herpes simplex virus, Type 2 E. Group A beta-hemolytic streptococcus Jacquet’s diaper dermatitis is a multifactorial process. Yeast, irritants and moisture all contribute to the occurrence of this eruption 68) The causative agent of Roseola is: A. A dsRNA virus B. A ssDNA virus C. Streptococcus D. A dsDNA virus Correct Choice E. A ssRNA virus Roseola is caused by Human Herpesvirus 6, a double stranded DNA virus 69) Which of the following is the most common complication associated with cutis marmorata telangectatica congenita? A. Systemic lupus erythematosus B. Hypercalcemia C. Limb hypertrophy or atrophy Correct Choice D. Atrial septal defect E. Seizure disorder 18
  • 19. Hypertrophy or atrophy of the affected limb is the most likely consequence of cutis marmorata telangectatica congenita. Orthopedic evaluation should be a part of the patient’s routine management 70) Which of the following may be associated? A. Atrial septal defect B. AVM C. Paronychia D. Seizure disorder Correct Choice E. Cleft palate Nevus sebaceus can very rarely be associated with multiple anomalies. Schimmelpenning syndrome can include seizure disorder, mental retardation, coloboma, as well as skeletal, cardiac and genitourinary abnormalities 71) A 4 month-old with difuse blisters and erosions has a skin biopsy diagnostic of generalized mastocytosis. Which topical dressing should be avoided in this patient? A. Petrolatum B. Mupirocin ointment C. Polymyxin B ointmentCorrect Choice D. Neomycin ointment E. Silver sulfadiazine Mastocytosis compromises a group of diseases characterized by increased number of mast cells in the skin and other organs. Seventy-five percent of cases occur before the age of 2. Patients with mastocytosis should avoid potential mast degranulators including aspirin, codeine, opiates, procaine, spicy foods, cheese, alcohol, polymyxin B. 72) A neonate presents with a large segmental hemangioma of the V1 distribution. What are features may be associated with this finding? A. Posterior fossa defectCorrect Choice B. Tram-track calcifications C. Muscular dystrophy D. M-paraproteinemia E. Anterior fossa defect PHACES syndrome consists of posterior fossa defect, hemangiomas (often segmental and large), arterial defects, cardiac defects, eye abnormaliteis, and sternal clefting. Posterior fossa defects include the Dandy-Walker malformtion 73) Late onset subungual keratotic tumors are associated with: A. Basal cell nevus syndrome B. Incontinentia pigmenti Correct Choice C. Neurofibromatosis Type 1 19
  • 20. D. Cowden syndrome E. Carney complex A NEMO gene defect can cause subungual keratotic growths. The typical age of presentation is early adulthood 74) A 2 week-old infant is brought to the ER with a rash on her face. She is found to have a 3rd degree heart block. What is the risk that a second child born to this mother will have the same diagnosis? A. 100% B. 10% C. 25%Correct Choice D. 5% E. 50% The diagnosis here is neonatal lupus erythematosus. Babies are normal at birth and develop skin lesions within the few months of life. About half of these babies will have an associated congenital heart block, usually 3rd degree, which is permanent. Most infants with NLE are girls and are born to mothers who are Ro/La positive. There is a 25% chance that a second child with have NLE 75) A newborn presents with a well-defined, shiny patch with complete alopecia on the vertex of the scalp along the suture lines. Which of the following is the first step in diagnosis? A. Calcium B. Skin biopsy C. Fungal culture D. MRI E. Skull x-ray Correct Choice The newborn most likely has aplasia cutis congenita. A skull x-ray would be the simplest, most cost effective means of identifying any underlying bony abnormality 76) Multiple lesions are associated with a defect in: A. Patched B. Ras C. Beta-catenin Correct Choice D. NEMO E. PTEN The bluish cystic nodule on the face of this boy is a pilomatrichoma. A defect in beta-catenin has been implicated 77) Which of the following disorders is more likely to occur in children with chronic fecal incontinence? A. Seborrheic dermatitis B. Perianal streptococcal disease 20
  • 21. C. Perianal pseudoverrucous papules and nodules Correct Choice D. Granuloma gluteale infantum E. Langerhans cell histiocytosis The warty papules of perianal pseudoverrucous papules and nodules are seen more commonly in the setting of chronic fecal incontinence 78) All four subtypes of Phakomatosis Pigmentovascularis have which feature in common: A. Nevus spilus B. Nevus anemicus C. Epidermal nevi D. Nevus flammeusCorrect Choice E. Dermal Melanocytosis. Phakomatosis Pigmentovascularis: Patients with a combination of vascular malformations and melanocytic or epidermal nevi are grouped into 4 subtypes of this disorder. All have nevus flammeus/capillary malformation (CM). Type I: CM + epidermal nevus Type II: CM + dermal melanocytosis +/- nevus anemicus Type III: CM + nevus spilus +/- nevus anemicus Type IV: CM + dermal melanocytosis + nevus spilus +/- nevus anemicus 79) The disorder caused by a defect in intestinal zinc-specific transporter SLC39A4 is most likely to present: A. At birth B. In adulthood C. Upon weaning from breast milk Correct Choice D. In childhood E. Upon weaning from formula In acrodermatitis enteropathica, there is decreased absorption of zinc from the infant’s gastrointestinal tract. The zinc in breast milk has greater availability than nonmaternal sources thus protecting the child from disease expression until weaning 80) The lesion depicted is most often associated with: A. Polyostotic fibrous dysplasia Correct Choice B. Chondrodysplasia punctata C. Cleft palate D. Osteopoikilosis E. Sphenoid wing hypoplasia The “coast of Maine” café au lait macule is associated with McCune-Albright syndrome and polyostotic fibrous dysplasia 81) What is the most likely diagnosis? 21
  • 22. A. German measles B. Letterer-Siwe disease C. Mucocutaneous lymph node syndrome D. Exanthem subitum E. Papular acrodermatitis of childhoodCorrect Choice Gianotti-Crosti syndrome is also known as papular acrodermatitis of childhood. The eruption is characterized by lichenoid papules in an acral distribution 82) A full term neonate is noted to have small pustules with no underlying erythema present at delivery. The pustules are easily removed with clearing of the vernix and a collarette appears. A gram stain is done showing predominately neutrophils without bacteria. What is the most likely diagnosis? A. Miliaria B. Transient neonatal pustular melanosis Correct Choice C. Urticaria pigmentosa D. Erythema toxicum neonatorum E. Congenital candidiasis Transient neonatal pustular melanosis typically begins with sterile pustules that leave a characteristic collarette when ruptured. The lesions heal with hyperpigmented macules 83) What is the most likely diagnosis: A. Facticial dermatosis B. Epidermolysis bullosa simplex Correct Choice C. Linear bullous IgA disease D. Epidermolysis bullosa dystrophica E. Pemphigus vulgaris The bullous lesions represented are nonscarring and are associated with obvious nail dystrophy. Epidermolysis bullosa simplex, Dowling-Meara type, is the best option 84) Which of the following diseases with immunodeficiency has an increased risk of lymphoreticular malignancy? A. Chronic granulomatous disease B. Severe combined immunodeficiency syndrome C. Leiner’s disease D. Wiskott-Aldrich syndromeCorrect Choice E. Job syndrome Wiskott-Aldrich syndrome is an X-linked recessive disorder caused by mutations in WAS gene. Patients with Wiskott-Aldrich have atopic dermatitis with increased risks for secondary infection, thrombocytopenia, and recurrent bacterial infections. They are also at an increased risk for lymphoreticular malignancy (20% 22
  • 23. 85) Which of the following is the most common long term sequelae from congenital rubella syndrome? A. Nystagmus B. Macrocephaly C. Microcephaly D. Deafness Correct Choice E. Saber shins Deafness may occur in up to 50% of infants with congenital rubella syndrome 86) Rhinorrhea, condylomata lata, and mucous patches are all seen with which congenital disorder? A. Human papillomavirus infection B. Herpes simplex virus C. Rubella D. Toxoplasmosis E. Syphilis Correct Choice Signs of congenital syphilis include rhinorrhea, snuffles, rhagades, condylomata lata, and mucous patches. Condylomata lata or a generalized papulosquamous eruption of secondary syphilis may be present in diaper area 87) What is the most likely diagnosis? A. Langerhans cell histiocytosis B. Perianal streptococcal disease Correct Choice C. Granuloma gluteale infantum D. Psoriasis E. Contact dermatitis The bright red erythema of perianal streptococcal disease can also involve the creases of the groin and axillae 88) What is the most likely diagnosis? A. Bullous pemphigoid of infancy B. Herpes zoster C. Epidermolysis bullosa simplex D. Incontinentia pigmenti Correct Choice E. Disseminated herpes simplex The vesicular lesions following the lines of Blaschko are typical for incontinentia pigmenti 89) Which of the following is most likely associated? A. No additional abnormality Correct Choice 23
  • 24. B. Alopecia C. Deafness D. Bifid rib E. Coarctation of the aorta The majority of patients with accessory tragii do not have any associated abnormalities. Multiple anomalies, including deafness and midline defects, have been reported but are rare 90) What is the most likely diagnosis? A. Seborrheic dermatitis B. Langerhans cell histiocytosis C. Psoriasis Correct Choice D. Perianal streptococcal disease E. Contact dermatitis The well-defined erythematous plaque is an example of psorisis in the diaper area. The moist nature of the diaper environment results in a loss of the classic silvery scale 91) A healthy, full-term infant develops a pustular, erythematous eruption on her face and trunk on the third day of life. A smear taken from one of these pustules would show: A. Predominantly neutrophils B. Predominantly eosinophilsCorrect Choice C. Gram-positive bacteria D. Hyphae E. Multi-nucleated giant cells This baby has developed erythema toxicum neonatorum, a benign, self-limited eruption that occurs in the majority of healthy, full-term infants. Erythema toxicum usually develops on the second or third day of life and resolves by day 10. A smear taken from one of the pustules will demonstrate eosinophils, which is sufficient to make the diagnosis. Viral infections of the skin may demonstrate multi-nucleated giant cells on Tzanck smear. Transient neonatal pustular melanosis is present at birth; neonates present with small, superficial pustules that rupture easily. Some may have ruptured in utero, leaving pigmented macules. The pigmentation may persist for weeks to months in darkly-complexed infants. A smear of the intracorneal/subcorneal pustule will show mostly neutrophils, but eosinophils may also be present 92) Which of the following clinical signs is most likely associated? A. Alopecia B. Hypertrichosis C. Clitoral hypertrophy Correct Choice D. Neurofibromas E. Axillary freckling The “coast of Maine” café au lait macule is associated with McCune-Albright syndrome. Endocrine abnormalities, including precocious puberty and polyostotic fibrous dysplasia are associated features. The genetic mutation which results in this genodermatosis is due to a post-somatic mutaion in the alpha sub-unit of stimulatory G-protein 24
  • 25. 93) A newborn infant presents with ring of long, dark, coarse hair surrounding a midline scalp patch of alopecia. What is associated with this finding? A. Thyroid disease B. Alopecia areata C. Ectopic brain tissueCorrect Choice D. Deafness E. Nevus sebaceus of Jadassohn The "hair collar sign" is associated with ectopic brain tissue and is thought to arise from a congenital herniation through the skull. Caution must be used in evaluation as biopsy or needle aspiration may lead to retrograde infection 94) What deficiency is responsible for this condition? A. Vitamin A B. Thiamine C. Vitamin C D. ZincCorrect Choice E. Niacin Acrodermatitis enteropathica is a rare inherited disorder transmitted in an autosomal recessive fashion. The disorder is caused by an inability to absorb zinc. the clinical syndrome is characterized by acral dermatitis, alopecia, and diarrhea. Zinc deficiency from other causes (including chronic wasting, poor oral intake, lack of supplementation in total parenteral nutrition) can cause similar clinical changes 95) The genetic disorder depicted in this photo is caused by a mutation in: A. STK11 Correct Choice B. Patched C. TSC1 D. PTEN E. MASH2 The multiple labial lentigo seen on this young boy are seen with Peutz-Jegher syndrome. A defect in the STK11 gene is responsible 96) A 2 year-old has a high fever for three days, as the fever breaks, a generalized rash appears on the trunk. The most likely diagnosis is: A. Rubella B. Erythema infectiosum C. Measles D. Roseola Correct Choice E. Mumps 25
  • 26. Roseola typically appears in toddlers. A high fever followed by a generalized maculopapular rash is characteristic for this HHV6 infection 97) What is the most likely neoplasm associated with this lesion? A. Keratoacanthoma B. Syringocystadenoma papilliferumCorrect Choice C. Clear cell acanthoma D. Basal cell carcinoma E. Pilomatricoma Nevus sebaceus is usually a solitary lesion that presents at birth or shortly after. It usually has a characteristic yellow or yellow brown color with a verrucous surface. Syringocystadenoma papilliferum is the most common neoplasm associated with nevus sebaceous 98) What is the most likely diagnosis is this 16 year old patient who developed generalized eruption 2 weeks after onset of sore throat due to strep infection? A. Lichen planus B. Cutaneous T-cell lymphoma C. Pityriasis rosea D. Psoriasis Correct Choice E. Contact dermatitis Although the exact mechanism is not known, there is a well-known correlation between a history of upper respiratory infection secondary to group A beta-hemolytic streptococci and the subsequent development of guttate psoriasis. The eruption may resolve spontaneously or with treatment (topical steroids or ultraviolet therapy). Some portion of these patients eventually develop chronic, plaque-type psoriasis 99) Psammomatous melanotic schwannomas are associated with: A. Tuberous sclerosis B. Neurofibromatosis Type 2 C. Bloom syndrome D. Carney complex Correct Choice E. Neurofibromatosis Type 1 Psammomatous melanotic schwannomas have been described in the Carney complex, a defect in the tumor suppressor gene, PRKAR1A 100) Rapp-Hodgkin is caused by a defect in the following gene: A. Ectodysplasin A B. P63Correct Choice C. Desmoglein 1 D. Plakophilin 26
  • 27. E. Connexin 30 Rapp-Hodgkin, along with the other ectodermal dysplasia-clefting syndromes, is reportedly caused by a defect in the p63 gene 101) The differential diagnosis of zinc deficiency is least likely to include: A. Cystic fibrosis B. Holocarboxylase synthetase deficiency C. Biotin deficiency D. Granuloma gluteale infantum Correct Choice E. Multiple carboxylase deficiency All of the options result in an eczematous acrodermatitis enteropathica-like eruption except granuloma gluteale infantum. As the name suggests, the lesions of granuloma gluteale infantum are granulomatous 27